Download as pdf or txt
Download as pdf or txt
You are on page 1of 78

CORRIGENDUM OF MOCK TEST 2

Due to typographical errors, there have been mistakes found in Mock Test 2. Please find the updated key
for Q.no- 96.

Q.96) With reference to the Asian Games 2023, consider the following statements :
1. It was held in Hangzhou in the People's Republic of China.
2. Cricket was included for the first time in this edition.
3. India earned its first badminton gold medal in this edition.
How many of the statements given above are correct ?
(a) Only one
(b) Only two
(c) All three
(d) None
EXPLANATION:
The 19th Asian Games 2023 was held in Hangzhou, People's Republic of China, from September 23 to
October 8, 2023.The Theme of the 19th Asian Games is "Heart to Heart, @Future".
The Asian Games are the oldest and most prestigious event on the Olympic Council of Asia (OCA) calendar,
dating back to the inaugural edition in 1951. Like the Olympic Games, they are held every four years. The
first edition of the Asian Games was held in New Delhi in March 1951.
So, Statement 1 is correct.
At the Asian Games, cricket made its debut as a medal event in 2010 and was contested again at the next
edition in 2014.Hangzhou 2023 also marked cricket’s return to the Asian Games programme after nine
years. After featuring at Guangzhou 2010 and Incheon 2014, the sport was dropped from the 2018 edition
in Jakarta. Like in 2010 and 2014, cricket at the Asian Games 2023 is being played in the T20 format.
Thus, Cricket was not included for the first time in this 2023 edition. So, Statement 2 is NOT correct.
Star men's doubles pair of Satwiksairaj Rankireddy and Chirag Shetty earned India's first-ever badminton
gold medal at the 19th Asian Games with a straight-game win over South Korea's Choi Solgyu and Kim
Wonho. So, Statement 3 is correct.

SIA-A-GS I (1-A)
ALL INDIA MOCK TEST III – 2024
EXPLANATION
INDEX
1. ............................................................................................................................................ 4
2. ............................................................................................................................................ 4
3. ............................................................................................................................................ 5
4. ............................................................................................................................................ 5
5. ............................................................................................................................................ 6
6. ............................................................................................................................................ 6
7. ............................................................................................................................................ 7
8. ............................................................................................................................................ 8
9. ............................................................................................................................................ 8
10. ............................................................................................................................................ 9
11. .......................................................................................................................................... 10
12. .......................................................................................................................................... 11
13. .......................................................................................................................................... 11
14. .......................................................................................................................................... 12
15. .......................................................................................................................................... 13
16. .......................................................................................................................................... 14
17. .......................................................................................................................................... 14
18. .......................................................................................................................................... 15
19. .......................................................................................................................................... 15
20. .......................................................................................................................................... 16
21. .......................................................................................................................................... 17
22. .......................................................................................................................................... 17
23. .......................................................................................................................................... 18
24. .......................................................................................................................................... 20
25. .......................................................................................................................................... 21
26. .......................................................................................................................................... 21
27. .......................................................................................................................................... 22
28. .......................................................................................................................................... 23
29. .......................................................................................................................................... 23
30. .......................................................................................................................................... 24
31. .......................................................................................................................................... 24
32. .......................................................................................................................................... 25

SIA-A-GS I (1-A)
33. .......................................................................................................................................... 26
34. .......................................................................................................................................... 27
35. .......................................................................................................................................... 27
36. .......................................................................................................................................... 28
37. ……….. ............................................................................................................................... 28
38. .......................................................................................................................................... 29
39. .......................................................................................................................................... 29
40. .......................................................................................................................................... 30
41. .......................................................................................................................................... 31
42. .......................................................................................................................................... 31
43. .......................................................................................................................................... 32
44. .......................................................................................................................................... 33
45. .......................................................................................................................................... 34
46. .......................................................................................................................................... 35
47. .......................................................................................................................................... 36
48. .......................................................................................................................................... 38
49. .......................................................................................................................................... 39
50. .......................................................................................................................................... 40
51. .......................................................................................................................................... 41
52. .......................................................................................................................................... 41
53. .......................................................................................................................................... 42
54. .......................................................................................................................................... 43
55. .......................................................................................................................................... 43
56. .......................................................................................................................................... 44
57. .......................................................................................................................................... 44
58. .......................................................................................................................................... 45
59. .......................................................................................................................................... 46
60. .......................................................................................................................................... 46
61. .......................................................................................................................................... 47
62. …………. .............................................................................................................................. 47
63. .......................................................................................................................................... 48
64. .......................................................................................................................................... 49
65. .......................................................................................................................................... 49
66. .......................................................................................................................................... 50
67. .......................................................................................................................................... 51

SIA-A-GS I (2-A)
68. .......................................................................................................................................... 51
69. .......................................................................................................................................... 52
70. .......................................................................................................................................... 53
71. .......................................................................................................................................... 53
72. .......................................................................................................................................... 54
73. .......................................................................................................................................... 54
74. .......................................................................................................................................... 55
75. .......................................................................................................................................... 56
76. .......................................................................................................................................... 56
77. .......................................................................................................................................... 57
78. .......................................................................................................................................... 58
79. .......................................................................................................................................... 59
80. .......................................................................................................................................... 60
81. .......................................................................................................................................... 60
82. .......................................................................................................................................... 61
83. .......................................................................................................................................... 62
84. .......................................................................................................................................... 62
85. .......................................................................................................................................... 63
86. .......................................................................................................................................... 63
87. .......................................................................................................................................... 64
88. .......................................................................................................................................... 65
89. .......................................................................................................................................... 65
90. .......................................................................................................................................... 65
91. .......................................................................................................................................... 66
92. .......................................................................................................................................... 67
93. .......................................................................................................................................... 67
94. .......................................................................................................................................... 68
95. .......................................................................................................................................... 68
96. .......................................................................................................................................... 70
97. .......................................................................................................................................... 70
98. .......................................................................................................................................... 71
99. .......................................................................................................................................... 71
100. .......................................................................................................................................... 72

SIA-A-GS I (3-A)
1. Consider the following pairs :
S.No Ancient Occupations
Terms
1. Langhika - Acrobats
2. Sokajjayika - Magician
3. Bhisakka - Physician
4. Ikkhanika - Women fortune-teller
How many above pairs are correctly matched?
(a) Only one
(b) Only two
(c) Only three
(d) All four
EXPLANATION:
The period c.600-300 BCE marks the beginning of the early historical period in north India. It was a major
landmark in Indian history which had far reaching consequences. For the first time in Indian history,
several territorial political entities emerged. These were called the mahajanapadas, and they spread over
most of north India. Cities and city life, which declined after the Harappan civilisation, emerged once again
along the Ganga Valley and spread to the far reaches of the northwest.
According to the narratives available from textual sources, cities often emerged from a group of villages
specialising in professions such as blacksmithing, pottery, carpentry, cloth-weaving, basket-weaving, etc.
Specialised craftsmen tended to congregate at places which offered either proximity to raw material (such
as clay for potters or timber for carpenters) or proximity and accessibility to market for distribution of their
crafted items. Buddhist sources, for example, speak appreciatively of distinctly urban occupations.
➢ Physicians (vejja, bhisakka)
➢ Surgeons (sallakata)
➢ Scribes (lekha)
➢ Accounting (ganana) and money changing were other urban occupations
➢ Some of them performed in fairs known as samajas, apart from other occasions. The Buddhist sources
in particular, show particular affinity for the accomplished courtesan (ganika), ambapali, to whom was
credited the prosperity of Vaishali.
➢ There were also a range of entertainers, known from both sanskrit and pali sources, like actors (nata),
dancers (nataka), magicians (sokajjayika), acrobats (langhika), drummers (kumbhathunika), and
women fortune-tellers (ikkhanika).
So, Option (d) is correct.

2. 'Paumacariyam' is a version of the epic Ramayana in which of the following religious sects of India?
(a) Buddhism
(b) Jainism
(c) Shaktism
(d) Charvaka
EXPLANATION:
The Jaina text of Paumacariyam is in Prakrit. It is a version of Ramayana. Vimalasuri, the author of
Paumacariyam, dates the text 530 years after the death of Mahavira. Some scholars date it slightly later
to the early centuries CE. One of the earliest Jaina versions, Paumacariyam, among others, was used to
propagate Jaina ethics. The story as claimed by Vimalasuri, is the same one as told by Mahavira to
Indrabhuti, which was then narrated by the latter to his disciples. The fact that the initial narration was
by a deity or an important person emphasises the importance of the text. The story is also called
Raghavacarita, and it is sometimes also called puranam akhyanam. Thus, the genre is similar to the
Brahmanical one. Paumacariyam is written in Prakrit and not Sanskrit. It thus had a wider reach. So,
Option (b) is correct.

SIA-A-GS I (4-A)
3. With reference to ancient religious beliefs, consider the following statements :
1. The term Ganadharas is associated with Buddhism
2. The doctrine of Syadvada in Jainism emphasises the relativity of all knowledge.
Which of the above statements is/are correct?
(a) 1 only
(b) 2 only
(c) Both 1 and 2
(d) Neither 1 nor 2
EXPLANATION:
Ganadhara refers to the "attendants of a Tīrthankaras" commonly found in Jaina iconography. Another
feature of the Jina icon is the presence of Ganadharas just to the right and left of the main figure. Jaina
texts (especially of iconography) mention them as attendants of a Tīrthankara. Such figures are
represented as holding some Chowris. One of them might be shown with its hands elapsed in adoration.
Though Yaksa figures are absent from the early Jina images of Mathura, Gaṇadhara figures are prominent
by their presence in many of these images. Hence, the term Ganadharas is not associated with Buddhism.
So, Statement 1 is not correct.
Jainism insists that every judgment (naya) should be qualified by some word like 'somehow' (syat, that is
to say, in some respect) so that the limitation of this judgment and the possibility of other alternative
judgments from other points of view may always be clearly borne in mind. The Jaina view that all our
ordinary knowledge is necessarily partial, being always relative to some particular point of view and about
particular aspects of objects, is known as syadvada, the doctrine of the relativity of knowledge and
judgements (syad: relative to some standpoint, vada: doctrine, theory, view). In terms of the seven-fold
formula, they supported their view that every proposition is only conditional and no proposition can be
either absolutely affirmed or absolutely negated. So, Statement 2 is correct.

4. Consider the following statements :


Statement I:
After the Independence of India, some of the princely states were reluctant to integrate with the
Union of India.
Statement II:
Mountbatten plan granted freedom to the Indian princely states to remain independent and not
to join with either country.
Which one of the following is correct in respect of the above statements?
(a) Both statement – I and Statement – II are correct and Statement – II is the correct explanation for
Statement –I.
(b) Both statement – I and Statement – II are correct and Statement – II is not the correct explanation
for Statement –I.
(c) Statement - I is correct but Statement - II is not correct.
(d) Statement - I is not correct but Statement – II is correct
EXPLANATION:
By August 15, 1947, all states except Kashmir, Hyderabad, and Junagarh had signed an instrument of
accession with the Indian government, acknowledging central authority over defence, external affairs, and
communication. These states sought various arrangements, including maintaining their sovereignty,
joining Pakistan, or remaining independent entities. The integration was achieved through a combination
of diplomatic negotiations, political agreements, and, in some cases, the use of force. So, Statement I is
correct.
Lord Mountbatten, India's last Viceroy, released a plan for the country's independence from the British on
June 3, 1947. The plan, also known as the Mountbatten Plan, came to be celebrated as the 'last plan for
independence'. Under this plan,
➢ Independence for princely states was ruled out, and they would join either India or Pakistan.
➢ Independence for Bengal was ruled out.
So, Statement II is not correct.
SIA-A-GS I (5-A)
5. ‘The Women, Business and Law 2024 report ’ is prepared by the
(a) World Bank
(b) International Alliance of Women
(c) UN Women
(d) International Federation of Business and Professional Women
EXPLANATION:
Women, Business and the Law 2024 report released by the World Bank is the 10th in a series of annual
studies measuring the enabling conditions that affect women’s economic opportunity in 190 economies. The
report covers 10 indicators: Safety, Mobility, Workplace, Pay, Marriage, Parenthood, Childcare,
Entrepreneurship, Assets, and Pension.
The report finds that women enjoy only two-thirds of the legal rights that men do and that the gender gap is
wider than laws on the books might suggest due to insufficient legal implementation.
India’s rank has marginally improved to 113, with a score of 74.4 per cent. While the country’s score has
remained constant since 2021, its ranking witnessed a decline from 122 in 2021 to 125 in 2022 and further
to 126 in the 2023 index.
Indian women have just 60 per cent of the legal rights compared to men, slightly below the global average of
64.2 per cent. India receives one of its lowest scores in the indicator evaluating laws impacting women’s pay.
So, Option (a) is correct.

6. In the context of Ecology, consider the following:


1. Bottom of the ocean
2. Cold deserts
3. Estuaries
4. Sites of volcanic eruption
In how many of the above ecologies, the ‘Edge effect’ is often witnessed?
(a) Only one
(b) Only two
(c) All three
(d) None
EXPLANATION:
Edge effects are the changes in biodiversity that occur inside the space surrounding the shared edge of
two or more distinct ecosystems. This transitional zone rich in biodiversity is known as the ecotone. An
ecotone is usually a more productive place than either of the individual ecozones because species of both
zones intermingle within it. Even in cases where there is less diversity, an ecotone is a good place for an
organism to be located as access to both ecozones is easier.
For example, Estuarine landscapes are comprised of mosaics of interconnected habitats such as mudflats,
salt marshes, shellfish reefs, mangrove forests, and seagrass meadows. Along the boundaries between
these habitats, edge effects may manifest as differences in the density, biomass, settlement, growth, or
survivorship of flora and fauna between the interior versus outer margin (i.e., edge) of a habitat patch.
So, Option (a) is correct.

SIA-A-GS I (6-A)
7. With reference to Hevea brasiliensis, sometimes seen in the news, consider the following statements:
1. It is the only cultivated commercial source of natural rubber.
2. It is native to the tropical evergreen rainforests of Southeast Asia.
Which of the above statements is/are not correct?
(a) 1 only
(b) 2 only
(c) Both 1 and 2
(d) Neither 1 nor 2
EXPLANATION:
Hevea brasiliensis is the major source of commercial natural rubber. Chemically, natural rubber is cis-1,4-
polyisoprene, having a molecular weight ranging from 200,000 to 8000,000 and with viscoelastic properties.
Natural rubber produced in the milky cytoplasm (latex) of specialized cells called laticifers is one of the most
important biological macromolecules, used as industrial raw material for the manufacture of about 50,000
products.
Although natural rubber has been found in the latex of over 2,000 plant species belonging to 311 genera of
79 families, H. brasiliensis remains the only cultivated species as a commercial source of natural rubber
because of its abundance in latex, high quality and convenience of harvesting.
So, Statement 1 is correct.
Hevea brasiliensis is a species of rubberwood that is native to rainforests in the Amazon region of South
America, including Brazil, Venezuela, Ecuador, Colombia, Peru, and Bolivia. These trees are generally found
in low-altitude moist forests, wetlands, riparian zones, forest gaps, and disturbed areas. It is a quick-
growing tree, often the first to establish itself when a gap in the canopy is produced, but may be shaded
out as more trees fill in the canopy opening. Therefore, Hevea brasiliensis is not native to southeast Asia
So, Statement 2 is not correct.

SIA-A-GS I (7-A)
8. Consider the following statements:
1. Oyster reefs are made up of numerous bivalve molluscs.
2. Oyster reefs usually do not coexist with mangroves.
3. Oyster reefs play a significant role in removing excessive nitrogen from aquatic ecosystems.
How many of the statements given above is/are correct?
(a) Only one
(b) Only two
(c) All three
(d) None
EXPLANATION:
An oyster reef is an ecological community made up of densely packed oysters (bivalve molluscs). It provides
a habitat for toadfish, skilletfish, and blue crabs. Oysters suck water through gills, and suspended tiny
organisms and pollutants like nitrogen are trapped in the gills' fine lining. They eat the organisms and expel
the rest as solid pellets of waste, which decompose and bubble up as nitrogen. A healthy oyster can filter
up to five litres of water per hour, thus cleansing it of algae and pollutants.
So, Statement 1 is correct.
Oyster reefs are mostly found with mangroves along the coast. Hence, they also defend shorelines and
reduce erosion. For example, Along the southwest Florida coast, oysters exist within the estuarine and
coastal areas as extensive reefs or isolated clusters or are attached to prop roots of red mangroves, often
extending out at the base of mangroves.
So, Statement 2 is not correct.

The biggest and most important benefit of oyster reefs to an ecosystem is their tremendous filtration
capacity. This filtration removes detritus, phytoplankton, contaminants, and bacteria, resulting in greater
light penetration, thus promoting the growth of seagrasses and benthic microalgae. Sequestration of
nitrogen via the removal of organic matter from the water column also decreases the impacts of
eutrophication and promotes denitrification. Therefore, Oyster reefs play a significant role in removing
excessive nitrogen from aquatic ecosystems. So, Statement 3 is correct.

9. With reference to Groundwater Nitrate contamination, consider the following statements:


1. Nitrate is very mobile and easily moves with water
2. Deeper groundwater & heavy clay soil led to increases in nitrate contamination.
3. Blue baby syndrome caused by nitrate contamination affects new-born babies' blood oxygen
4. Boiling the water reduces nitrate contamination
SIA-A-GS I (8-A)
How many of the statements given above is/are correct?
(a) Only one
(b) Only two
(c) Only three
(d) All four
EXPLANATION:
Nitrate is the oxidised form of dissolved nitrogen, which is the main source of nitrogen for plants. It occurs
naturally in soil and dissipates when the soil is extensively farmed. Thus, nitrogen fertilizers are applied to
replenish the soil. However, these nitrates can be toxic, especially when they enter the food chain via
groundwater and surface water. Nitrates may originate from manures, the decay of plants and other organic
materials or from fertilisers.
Nitrate is very mobile and is easily moved by water. When applied to the substrate, it moves readily with
water and does not attach to substrate particles. This mobility can have advantages in some situations but
can also result in nitrate leaching and loss. Other forms of nitrogen, such as ammonium (NH 4+), generally
do not leach. Unlike nitrate, the ammonium form does not leach because it is attached to the soil and resists
movement with water.
So, Statement 1 is correct.
Heavy rains can cause nitrates to leach downward in the soil below the root zone. Whether nitrates continue
to leach downward and into groundwater depends on underlying soil and bedrock conditions, as well as the
depth of groundwater. If the depth to groundwater is shallow and the underlying soil is sandy, the potential
for nitrates to enter groundwater is relatively high.
However, if the depth to groundwater is deep and the underlying soil is heavy clay, nitrates will not likely
enter groundwater. In some cases where dense hardpans are present, nitrate leaching will not progress
beyond the depth of the hardpan.
So, Statement 2 is not correct.
Once nitrates get into the groundwater, the greatest concerns are for infants less than one-year-old and
young or pregnant animals. High levels of nitrates can be toxic to newborns, causing anoxia or internal
suffocation.
The most common symptom of nitrate poisoning in babies is a bluish colour to the skin, particularly around
the baby's eyes and mouth. These symptoms of nitrate toxicity are commonly referred to as the "blue-baby"
syndrome. This occurs due to a decreased amount of haemoglobin (Blood protein that is responsible for
carrying oxygen around the body) in the baby’s blood. Therefore, Blue baby syndrome caused by nitrate
contamination affects newborn babies' blood oxygen. So, Statement 3 is correct.
Boiling water will not reduce nitrate levels. In fact, it will make the level of nitrate increase because some of
the water will evaporate, but the nitrate will not. This increases the concentration of nitrate in the water.
The potential of nitrates from animal manure and nitrogen fertilisers getting into groundwater can be
reduced through good management practices. However, little can be done to minimise the leaching of
nitrates into groundwater that results from the ongoing decay of organic matter in the soil. In this situation,
nitrates are simply a natural biological result of an ongoing cycle of nitrogen transformation processes in
the soil. So, Statement 4 is not correct.

10. In an economy, if there is a sudden and substantial inflow of foreign investments due to reforms
initiated by the Government. Which of the following is/are potential outcomes?
1. Excessive appreciation of domestic currency
2. Loss of export competitiveness
3. Preference for import commodities over domestically produced ones
Select the correct code given below:
(a) 1 and 2 only
(b) 3 only
(c) 1, 2 and 3
(d) None
SIA-A-GS I (9-A)
EXPLANATION:
Many developing countries have reaped handsome rewards from surging capital inflows in recent years.
This is widely regarded as a very welcome phenomenon, raising levels of investment and encouraging
economic growth. But surging capital inflows can also be something of a double-edged sword, inflicting
rather less welcome and destabilising side effects, including a tendency for the local currency to gain in
value, undermining the competitiveness of export industries and potentially giving rise to inflation.
Currency appreciation refers to an increase in the value of one currency in relation to another currency. It
occurs when the exchange rate for a currency rises over time. Simultaneously, the currency
appreciation benefits importers as they have to pay less in domestic currency for imported goods. Due to
this, the cost of imported goods has become less, so people prefer to import commodities over domestically
produced ones.
Capital inflows result in a build-up of foreign exchange reserves. As these reserves are used to buy domestic
currency, the domestic monetary base expands without a corresponding increase in production: too much
money begins to chase too few goods and services. So, Statements 1, 2 and 3 are correct.

11. Consider the following :


1. Regulating stock brokers
2. Promoting investors' education
3. Promoting insider trading in securities
4. Regulating substantial acquisition of shares and takeover of companies
How many of the above statements is/are not correct with respect to the functions of the Securities
and Exchange Board of India (SEBI)?
(a) Only one
(b) Only two
(c) Only three
(d) All four
EXPLANATION:
The provisions of the Securities and Exchange Board of India Act, 1992 it shall be the duty of the SEBI to
protect the interests of investors in securities and to promote the development of, and to regulate the
securities market, by such measures as it thinks fit. Without prejudice to the generality of the foregoing
provisions, the measures referred to therein may provide for:
➢ Regulating the business in stock exchanges and any other securities markets;
➢ Registering and regulating the working of stock brokers, sub-brokers, share transfer agents,
bankers to an issue, trustees of trust deeds, registrars to an issue, merchant bankers, underwriters,
portfolio managers, investment advisers, and such other intermediaries who may be associated with
securities markets in any manner. So, Statement 1 is correct.
➢ Registering and regulating the working of the depositories, participants, custodians of securities,
foreign institutional investors, credit rating agencies, and such other intermediaries as the Board
may, by notification, specify on this behalf;
➢ Registering and regulating the working of venture capital funds and collective investment schemes,
including mutual funds;
➢ Prohibiting fraudulent and unfair trade practices relating to securities markets;
➢ Promoting investors' education and training of intermediaries of securities markets; So, Statement
2 is correct.
➢ Safeguards investors’ interests by monitoring and regulating the securities market, and it does not
promote insider trading. Insider trading involves trading in a public company's stock or other
securities by someone with non-public, material information about the company. Insider
transactions are legal if the insider makes a trade and reports it to the Securities and Exchange
Commission, but insider trading is illegal when the material information is still non-public. So,
Statement 3 is not correct.
➢ Regulating substantial acquisition of shares and take-over of companies. So, Statement 4 is
correct.

SIA-A-GS I (10-A)
12. Consider the following statements about the Fiscal Responsibility and Budget Management (FRBM)
Act
Statement I :
In spite of the severe financial stress caused by the COVID-19 pandemic, the government refrained
from monetising the deficit.
Statement II :
Monetising deficit could have adverse consequences, including the potential to cause
hyperinflation in an economy.
(a) Statement I and Statement II are correct and Statement II is the correct explanation for
Statement I.
(b) Statement I and Statement II are correct, but Statement II is not an explanation got Statement I.
(c) Statement I is correct but Statement II is incorrect.
(d) Statement I is incorrect but Statement II is correct.
EXPLANATION:
The FRBM act pointed that the Central Government shall not borrow from the Reserve Bank of India. But
the RBI could subscribe to the primary issue of central government securities in case the government
exceeds the fiscal deficit target on “grounds of national security, act of war, national calamity, collapse of
agriculture severely affecting farm output and incomes.
The covid 19 pandemic had meant a sharp rise in government expenditures even as revenue collections
plunge, exerted pressure on the rising fiscal deficit and India’s economy was expected to grow at less than
5 per cent in 2019-20 and at 2 per cent or less in 2020-21 due to a complete halt in economic activity.
Eventhough, the economic situation was bad, the government opted out from monetizing the fiscal deficits
to avoid high inflationary risks. So, Statement I is correct.
Monetising fiscal deficit means the RBI purchases government debt directly rather than the government
borrowing from the markets by selling bonds. In turn, the central bank prints more currency to finance
this debt. This increases the money supply in the economy leading to inflationary pressures.
When the government spends the extra funds that have come into its account, there is an increase in
‘Base money’, that is, currency plus banks’ reserves. So, monetisation results in an expansion of money
supply. The primary concern about engaging in monetization is the fear that it will lead to excessive and
uncontrollable inflation.
So, Statement II is correct.
Statement I and Statement II are correct and Statement II is the correct explanation for
Statement I.

13. Consider the following statements:


1. The same good could be an intermediate good or a final good, depending on use.
2. The same good could be a consumer good or a capital good, depending on its use.
3. The same good could be a fast-moving consumer good or a durable consumer good based on use.
Which of the above statements are correct?
(a) 1 and 2 only
(b) 2 and 3 only
(c) 1 and 3 only
(d) 1, 2 and 3
EXPLANATION:
An intermediate good is a product used to produce a final good or finished product also referred to as a
consumer good.
Goods which are used either for final consumption by the consumers or for investment by the producers
are known as final goods. Intermediate goods like salt can also be finished products, since it is consumed
directly by consumers and used by producers to manufacture other food products. So, the same good
could be an intermediate good or a final good, depending on use. So, Statement 1 is correct.

SIA-A-GS I (11-A)
Capital goods and consumer goods are terms used to describe goods based on their use. A capital good is
any physical asset used for production (by businesses to produce goods or services for consumers).
Consumer goods are those used by consumers and have no future productive use.
The same physical good could be either a consumer or capital good, depending on how the good is used.
For example, a riding lawn mower purchased by a landowner to mow the yard is a consumer good; the
same lawn mower purchased by a lawn care business is a capital good. So, Statement 2 is correct.
Fast-moving consumer goods are products that sell quickly at relatively low cost. These goods are also
called consumer packaged goods or non durable consumer goods. FMCGs have a short shelf life because
of high consumer demand (e.g., soft drinks and confections) or because they are perishable (e.g., meat,
dairy products, and baked goods).
Durable consumer goods are products which are used for a long period. Examples of such goods are
refrigerator, car, washing machine etc. Durable goods have a shelf life of three years or more, while non-
durable goods have a shelf life of less than three years. The same good could not be a fast-moving consumer
good or a durable consumer good based on use because, unlike FMCGs, durable consumer goods are not
consumed or used up quickly and are intended for long-term use. So, Statement 3 is not correct.

14. Which among the following initiatives of NITI Aayog promotes competitive federalism in our
country?
1. India Innovation Index
2. School Education Quality Index
3. Subgroup of Chief Ministers
4. Aspirational Districts programme
Select the correct answer from the codes given below :
(a) 1 and 2 only
(b) 3 and 4 only
(c) 1, 2 and 4 only
(d) 1, 2, 3 and 4
EXPLANATION:
“Competitive federalism” refers to the existence and desirability of competition among governments and
jurisdictions in a federal political system.
India opted for a quasi-federal structure after independence. The term “federal” has not been mentioned in
the constitution but the working of Indian democracy is essentially federal in structure. In Competitive
federalism, the relationship between the Central and state governments is vertical, and between state
governments is horizontal. NITI Aayog endeavours to promote competitive federalism by facilitating
improved performance of States/UTs. It encourages healthy competition among states through transparent
rankings in various sectors, along with a hand-holding approach. Some of the indices launched by NITI
Aayog are
➢ School Education Quality Index, So, Statement 2 is correct.
➢ State Health Index,
➢ Composite Water Management Index,
➢ Sustainable Development Goals Index,
➢ India Innovation Index and Export Competitiveness Index. So, Statement 1 is correct.
➢ NITI Aayog also releases delta rankings for the performance of Aspirational Districts every month
through the Aspirational Districts Programme. The ranking of States in various social sectors based on
quantitative objective criteria encourages them, and even districts, to improve their performance. NITI
Aayog works closely with all stakeholders, including the State/UT Governments and concerned
Ministries/Departments, in developing indicator frameworks, review mechanisms and capacity-
building. So, Statement 4 is correct.
➢ The subgroup of Chief Ministers comes under cooperative federalism. Cooperative federalism is a model
of intergovernmental relations that recognizes the overlapping functions of the national and state

SIA-A-GS I (12-A)
governments. This model can be contrasted with the model of dual federalism, which maintains that
the national and state governments have distinct and separate government functions. So, Statement
3 is not correct.

15. Consider the following:


1. National Commission for Women
2. NITI Aayog
3. The Comptroller and Auditor General of India
4. Union Public Service Commission
How many of the above are quasi-judicial bodies?
(a) Only one
(b) Only two
(c) Only three
(d) All four
EXPLANATION:
When a body has judicial authority but only partially exercises it, it is referred to as a quasi-judicial body.
It performs some of the functions of a court, including adjudication and decision-making, but it is not
strictly obligated to follow those procedures.
The National Commission for Women was set up as a statutory body in 1992 under the National
Commission for Women Act, 1990 to review the Constitutional and legal safeguards for women, recommend
remedial legislative measures, facilitate redressal of grievances and advise the Government on all policy
matters affecting women.
The commission looks into complaints and takes suo moto notice. The Commission, while investigating any
matter or inquiring into any complaint, has all the powers of a civil court trying a suit in particular. Hence,
the National Commission for Women is a quasi-judicial body. So, Statement 1 is correct.
NITI Aayog, like that of the Planning Commission, was also created by an executive resolution of the
Government of India. Hence, it is also neither a constitutional body nor a statutory body. In other words, it
is a non-constitutional or extra-constitutional body.
NITI Aayog is the premier policy ‘Think Tank’ of the Government of India, providing both directional and
policy inputs. While designing strategic and long-term policies and programmes for the Government of
India, NITI Aayog also provides relevant technical advice to the Centre and States. Niti Ayog is not a quasi-
judicial body. So, Statement 2 is not correct.
The Constitution of India (Article 148) provides for an independent office of the Comptroller and Auditor
General of India (CAG). He is the guardian of the public purse and controls the entire financial system of
the country at both levels–the Centre and the State. His duty is to uphold the Constitution of India and the
laws of Parliament in the field of financial administration. CAG’s Audit in relation to the regularity of
expenditure is quasi-judicial. It involves interpretation of the Constitution, Statutes, rules and orders with
reference to the case law of previous decisions and precedents.
➢ Quasi-judicial functions: This is a settled issue within the Department that the orders passed by
regulatory bodies in the exercise of their quasi-judicial functions, as its legality and justice ability, shall
not be within the scope of the audit.
➢ Audit around the quasi-judicial functions: This includes the process of reaching decisions, availability
of all required information and implementation of decisions, excluding the legality and justice ability.
So, Statement 3 is correct.
The Union Public Service Commission (UPSC) is the central recruiting agency in India. It is an independent
constitutional body in the sense that the Constitution has directly created it. Articles 315 to 323 in Part
XIV of the Constitution contain elaborate provisions regarding the composition, appointment and removal
of members, along with the independence, powers and functions of the UPSC.
The quasi-judicial jurisdiction of the UPSC is limited both in scope and extent. In fact, it has no true
appellate jurisdiction. It can only advise on disciplinary actions taken against employees. Hence, UPSC is
not a quasi-judicial body. So, Statement 4 is not correct.
SIA-A-GS I (13-A)
16. Promotion of feelings of enmity or hatred between different classes of the citizens of India, on the
grounds of religion, is a:
(a) ‘Reasonable restriction’ under Article 19 of the Constitution of India
(b) 'Scheduled offence' under the Prevention of Money Laundering Act of 2002
(c) 'Scheduled offence' under the Unlawful Activities (Prevention) Act of 1967
(d) ‘Corrupt practice’ under the Representation of People Act of 1951
EXPLANATION:
Section 123 of the Representation of People Act of 1951 defines 'corrupt practices' to include bribery, undue
influence, false information, and promotion or attempted promotion of "feelings of enmity or hatred between
different classes of the citizens of India on the grounds of religion, race, caste, community, or language" by
a candidate for the furtherance of his prospects in the election. Under the provisions of the Act, an elected
representative can be disqualified if convicted of certain offences on the grounds of corrupt practices, for
failing to declare election expenses, and for interests in government contracts or works.
So, Option (d) is correct.

17. Consider the following:


1. Cut Motions in the budget
2. Calling Attention Motion
3. Adjournment Motion
4. Censure Motion
5. Motion of Thanks to the President’s Address
In how many of the above instruments of the Parliament does the legislature exercise control over
the executive?
(a) Only two
(b) Only three
(c) Only four
(d) All five
EXPLANATION:
The Constitution of India established a parliamentary form of government in which the Executive is
responsible to the Parliament for its policies and acts. Hence, the Parliament exercises control over the
Executive through question-hour, zero-hour, half-an-hour discussions, short-duration discussions, calling
attention motions, adjournment motions, no-confidence motions, censure motions and other discussions.
➢ A cut motion is a special power vested in members of the Lok Sabha to oppose a demand being discussed
for specific allocation by the government in the Finance Bill as part of the Demand for Grants. So,
Statement 1 is correct.
➢ Calling Attention Motions: A member may, with the prior permission of the Speaker, call the attention
of a Minister to a matter of urgent public importance and request him to make a statement thereon. So,
Statement 2 is correct.
➢ The adjournment motion is introduced in the Parliament to draw the attention of the House to a definite
matter of urgent public importance. It needs the support of 50 members to be admitted. So, Statement
3 is correct.
➢ "Censure Motion" - A motion moved against the Government censuring its policy in some direction or
an individual minister or ministers of the Government. So, Statement 4 is correct.
➢ The Lok Sabha can also express a lack of confidence in the government in the following ways:
1) By rejecting a money bill.
2) By defeating the government on a vital issue.
3) By not passing a motion of thanks on the President’s inaugural address.
So, Statement 5 is correct.

SIA-A-GS I (14-A)
18. Consider the following statements :
1. Fundamental Rights guaranteed under Article 19 and Article 21 are horizontally enforceable by a
court of law.
2. While most fundamental rights provide a negative function to the State, some offer a positive
function to the State institutions.
Which of the above statements is/are not correct ?
(a) 1 only
(b) 2 only
(c) Both 1 and 2
(d) Neither 1 nor 2
EXPLANATION:
The Supreme Court in Kaushal Kishore v. Union of India, with a majority vote of 4:1, ruled that
fundamental rights guaranteed under Article 19 and Article 21 of the Indian Constitution are horizontally
enforceable. Rights are said to be horizontally enforceable when they can be judicially enforced against
private parties by a court of law. The scheme of Part III of the Constitution of India defines both the State
(under Article 12) and law (under Article 13 Clause 3) for the purpose of vertical enforceability of the rights.
Clause 3 of Article 13 in its ambit of law includes laws made by parliament, state legislatures, ordinances
promulgated by the President or governors, bye-laws, rules, regulations, notifications, or customs or
usages. Both Articles 12 and 13 together envisage that the actor has to be the State/government in case
of a violation of fundamental right. So, Statement 1 is correct.
The fundamental rights are the negative in character, which places limitations on the authority of the
State. Examples are,
➢ Article 14 – The State shall not deny to any person equality before the law or the equal protection of
the laws within the territory of India.
➢ Article 15(1) – The State shall not discriminate against any citizen on grounds only of religion race,
caste, sex, place of birth or any of them.
➢ Article 16(2) – No citizen can be ineligible for or discriminated against any employment or office under
the State. These Articles place some limitations on the authority of the State.
The other Fundamental Rights which are positive in nature are Article 25, Article 29(1), Article 30(1) etc.),
conferring certain privileges on the persons.
➢ Article 21A - The State shall provide free and compulsory education to all children of the age of six to
fourteen years.
➢ Article 25 – All persons are equally entitled to freedom of conscience and the right to profess, practice
and propagate religion freely. These Articles confer certain privileges on the perons.
Therefore, Most of the Fundamental rights provide a negative function to the State, while other
Fundamental Rights provide a function to the State institutions. So, Statement 2 is correct.

19. Which of the following statements about the Panchayati Raj Institutions (PRIs) in India is not correct?
(a) Gram Sabha is a body consisting of persons registered in the electoral rolls of a village comprised
with in the area of Panchayat at the village level.
(b) Subject to a population limit as specified in the Constitution, PRIs in a state can either be a two
or three-tier system.
(c) The chairperson of a panchayat at a village level shall be elected in a manner determined by the
state legislature.
(d) A PRI can be dissolved before the completion of its term of five years for the specific reasons
as provided in the Constitution.
EXPLANATION:
The 73rd Amendment Act of 1992 provides for a Gram Sabha as the foundation of the Panchayati Raj
system. It is a body consisting of persons registered in the electoral rolls of a village comprised within the
area of Panchayat at the village level. Thus, it is a village assembly consisting of all the registered voters

SIA-A-GS I (15-A)
in the area of a panchayat. It may exercise such powers and perform such functions at the village level as
determined by the State Legislature. So, Option (a) is correct.
The 73rd Amendment Act of 1992 provides for a three tier system of Panchayat Raj in every state (Village
intermediate and district levels). The Act brings the uniformity in the structure of panchayati raj
throughout the country. However, a state having a population not exceeding 20 lakh may not constitute
panchayats at the intermediate level. Therefore, Panchayat Raj Institutions in a state can either be a two
or three-tier system, subject to a population limit as specified in the Constitution. So, Option (b) is
correct.
The Chairperson of the panchayat at the village level shall be elected in such manner as determined by
the State Legislature. However, the chairperson of panchayats at the intermediate and district levels shall
be elected indirectly by and from amongst the elected members of the panchayat.
The chairperson of a panchayat and other members of a panchayat elected directly or indirectly shall have
the right to vote in the meetings of the panchayats. So, Option (c) is correct.
The 73rd Amendment Act of 1992 provides for a five-year term of office to the panchayat at every level from
the date of its first meeting. However, it can be dissolved before the completion of its term. Further fresh
elections to constitute a panchayat shall be completed, Before the expiry of its duration of five years (or)
In case of dissolution, before the expiry of a period of six months from the date of its dissolution. But,
where the remainder of the period is less than six months, it shall not be necessary to hold any election
for constituting the new panchayat for such a period.
Article 243E of the Indian constitution deals with the dissolution of the panchayats but there is no specific
reason for the dissolution is mentioned in the Constition. So, Option (d) is not correct.

20. Consider the following statements with respect to the Ganges River system :
1. Rapti, Sarju and Sarda are tributaries of the Ghaghara River.
2. Chambal, Betwa, and Ken are tributaries of Yamuna River.
3. Mayurakshi, Damodar and Haldi are tributaries of the Hooghly River.
Which of the statements given above are correct?
(a) 1 and 2 only
(b) 2 and 3 only
(c) 1 and 3 only
(d) 1, 2 and 3
EXPLANATION:
Ghaghra is the Left Bank Tributaries of The Ganga River. River Ghaghara originates from the glaciers of
Mapchachungo. Its catchment area is 127,950 square kilometres, out of which only 45 per cent lies in
India. After joining the waters of the Beri, Tila and Seti rivers, Ghaghara carves out a deep gorge of about
600 metres deep at Shishapani. River Sarda joins it in the plain and further flows in south eastwardly
direction before finally merging into the Ganga at Chhapra.
Its important tributaries are the Sarda, the Sarju (Ayodhya is located on its bank) and the Rapti. The
Ghaghara joins the Ganga a few kilometres downstream of Chhapra in Bihar. So, Statement 1 is correct.
River Yamuna, with its length of about 1,384 kilometres, rises from Yamunotri glacier on the western slope
of Bandarpunch range at an elevation of 6313 metres. Its water is important for the eastern and western
Yamuna canals and Agra canals for irrigation. It runs parallel to river Ganga and joins it at Prayag in
Allahabad.
Its main southern tributaries are Ken, Betwa, Sind, and Chambal, whereas Sasur Khaderi, Sengar,
Hindon, Tons, Baruna and Rind are its northern tributaries. So, Statement 2 is correct.
The Hooghly River, also known as the Ganga or Kati-Ganga in the Puranas, is the western distributary of
the Ganges. It rises close to Giria, north of Baharampur and Palashi in Murshidabad. The river provides
access to Kolkata from the Bay of Bengal and is formed by the junction of the Bhagirathi and Jalangi rivers
at Nabadwip.
The tributaries of the Hooghly River are Mayurakshi, Jalangi, Ajay, Damodar, Rupnarayan, and Haldi
rivers. So, Statement 3 is correct.

SIA-A-GS I (16-A)
21. Consider the following statements with respect to the state of Manipur :
1. It is bordered by four Indian states.
2. Barak is the largest river of Manipur and it drains into the Loktak lake.
3. National Highway 2 connects Manipur with the rest of India.
How many of the statements given above are correct?
(a) Only one
(b) Only two
(c) All three
(d) None
EXPLANATION:
Manipur is one of the Border States in the north-eastern part of the country, having an international
boundary of about 352 km Long stretch of land with Myanmar in the southeast. It is bounded by Nagaland
in the north, Assam in the west and Mizoram in the south. It has a total area of 22327 sq. km. It lies
between 23.80 N to 25.70 N latitude and 93.50 E to 94.80 E longitudes. Thus, Manipur shares its borders
with three states. So, Statement 1 is not correct.

The Barak River is the largest river in Manipur, flowing for 900 kilometres through the states of Manipur,
Nagaland, Mizoram, and Assam. It flows into Bangladesh, where it bifurcates into the Surma River and the
Kushiyara River, which converge again to become the Meghna River before forming the Ganges Delta. Thus,
it does not drain into the Loktak lake. So, Statement 2 is not correct.
Land-locked Imphal is connected to the rest of India by two key national highways – the National Highway
No. 2 that passes through its Kangpokpi district to Nagaland and the National Highway 37 (now renamed
NH 127) through Jiribam to Assam’s Barak valley. National Highway 2 (NH-2) is a roadway linking Assam
with neighbouring states like Nagaland, Manipur, and Mizoram. It is a part of Asian Highways 1 and 2,
which runs from Imphal to Kohima. NH-2 has emerged as a transformative force in enhancing inter and
intra-state connectivity in the region. So, Statement 3 is correct.

22. Consider the following statements with respect to seismic waves :


1. P-waves are faster and can travel through solids, liquids, and gases.
2. S-waves cause particles to vibrate in the same direction as the wave propagates.
3. The shadow zone of S-waves is larger than the P-waves.
SIA-A-GS I (17-A)
Which of the following statements are correct?
(a) 1 and 2 only
(b) 2 and 3 only
(c) 1 and 3 only
(d) 1, 2 and 3
EXPLANATION:
Earthquake waves are basically of two types, namely, body waves and surface waves. Body waves are
generated due to the release of energy at the focus and move in all directions travelling through the body
of the earth.
There are two types of body waves. They are called P and S-waves. P-waves move faster and are the first
to arrive at the surface. These are also called 'primary waves'. The P waves are similar to sound waves.
They travel through gaseous, liquid and solid materials. So, Statement 1 is correct.
Different types of earthquake waves travel in different manners. As they move or propagate, they cause
vibration in the body of the rocks through which they pass. P-waves vibrate parallel to the direction of the
wave.
S-waves arrive at the surface with some time lag. These are called secondary waves. An important fact
about S-waves is that they can travel only through solid materials. The direction of vibrations of S-waves
is perpendicular to the wave direction in the vertical plane. Thus, S-waves do not cause particles to vibrate
in the same direction as the wave propagation. So, Statement 2 is not correct.
Earthquake waves get recorded in seismographs located at far off locations. However, there exist some
specific areas where the waves are not reported. Such a zone is called the 'shadow zone'.
It was observed that seismographs located at any distance within 105° from the epicentre recorded the
arrival of both P and S-waves. However, the seismographs located beyond 145° from the epicentre record
the arrival of P-waves but not that of S-waves. Thus, a zone between 105° and 145° from the epicentre was
identified as the shadow zone for both types of waves. The entire zone beyond 105° does not receive S-
waves.
The shadow zone of the S-wave is much larger than that of the P-wave. The shadow zone of P-waves
appears as a band around the earth between 105° and 145° away from the epicentre. The shadow zone of
S-waves is not only larger in extent, but it is also a little over 40 per cent of the earth's surface. So,
Statement 3 is correct.

23. Consider the following statements regarding the natural vegetation of India :
1. The Western Ghats are known for their evergreen forests, while the Eastern Ghats primarily
consist of deciduous forests.
2. The Deccan Plateau is characterised by thorn and scrub vegetation due to its semi-arid climate.

SIA-A-GS I (18-A)
3. The Himalayan region exhibits a vertical zonation of vegetation, with tropical deciduous forests at
lower altitudes and alpine vegetation at higher altitudes.
4. The Gangetic plains are predominantly covered by tropical rainforests owing to their fertile alluvial
soil.
Which of the statements given above are correct?
(a) 1 and 2 only
(b) 2 and 3 only
(c) 1, 2, and 3 only
(d) 1, 3, and 4 only
EXPLANATION:
The mountain chain of the Western Ghats represents geomorphic features of immense importance with
unique biophysical and ecological processes. The forests of the site include some of the best representatives
of non-equatorial tropical evergreen forests anywhere and are home to at least 325 globally threatened flora,
fauna, bird, amphibian, reptile and fish species.
The forests of Eastern Ghats include dry evergreen ones, semi-evergreen forests, southern tropical dry
mixed deciduous forests, dry savannah forests, southern tropical dry scrub forests, southern tropical thorn
forests, Carnatic umbrella thorn Forests, southern sub-tropical hill forests and southern thorn scrub. Thus,
Eastern Ghats primarily consist of deciduous forests. So, Statement 1 is correct.

SIA-A-GS I (19-A)
The Deccan Plateau experiences relatively low rainfall compared to other regions in India, leading to the
dominance of hardy vegetation such as thorny shrubs, bushes, and drought-resistant grasses.
The tropical scrubland forests in India’s Deccan Plateau region are commonly known as “dry deciduous
forests” or “Deccan thorn scrub forests.” These forests are characterized by a mix of short, thorny trees,
shrubs, and bushes that have adapted to survive in semi-arid and arid conditions. They are found in the
drier parts of the Deccan Plateau, which includes parts of Maharashtra, Karnataka, Telangana, and Andhra
Pradesh. Thus, The Deccan Plateau is characterized by thorn and scrub vegetation due to its
semi-arid climate. So, Statement 2 is correct.
The vegetation in the Himalayas varies greatly with altitude, climate, and topography. The Himalayan region
harbours a rich biodiversity, with numerous endemic plant species adapted to the diverse climatic
conditions across various elevations. In mountainous areas, the decrease in temperature with increasing
altitude leads to the corresponding change in natural vegetation.
The Tropical deciduous forests are the type of vegetation found in areas receiving annual rainfall of 100 to
200 cms. These include the Sahyadris, the northeastern plateau of the peninsula, the Himalayan foot hills
in the Siwaliks, the bhabars and terai.
The wet temperate types of forests are found between a height of 1000 and 2000 metres. Between 1500 and
3000 metres, temperate forests containing coniferous trees, like pine, deodar, silver fir, spruce and cedar,
are found. These forests cover mostly the southern slopes of the Himalayas, places having high altitudes in
southern and north-east India. At higher elevations, temperate grasslands are common. At high altitudes,
generally more than 3,600 metres above sea level, temperate forests and grasslands give way to the Alpine
vegetation. Silver fir, junipers, pines and birches are the common trees of these forests. However, they get
progressively stunted as they approach the snow line. Ultimately, through shrubs and scrubs, they merge
into the Alpine grasslands. These are used extensively for grazing by nomadic tribes like the Gujjars and
the Bakarwals. At higher altitudes, mosses and lichens form part of tundra vegetation. Thus, The
Himalayan region exhibits a vertical zonation of vegetation, with tropical deciduous forests at lower altitudes
and alpine vegetation at higher altitudes. So, Statement 3 is correct.
The Ganga plain extends between the Ghaggar and Teesta rivers. It is spread over North India in Haryana,
Delhi, U.P., Bihar, partly Jharkhand and West Bengal. In the East, particularly in Assam lies the
Brahmaputra plain. The Gangetic plains in India are not predominantly covered by tropical rainforests.
Instead, they are characterised by fertile alluvial soil and are primarily covered by various types of
agricultural land, including rice paddies, wheat fields, and other crops. Tropical rainforests are typically
found in regions closer to the equator with high rainfall and consistently warm temperatures throughout
the year. The Gangetic plains, although receiving significant rainfall during the monsoon season, do not
exhibit the characteristics of tropical rainforests. So, Statement 4 is not correct.

24. "Valparaiso", a region that recently experienced severe destruction due to a forest fire, is situated in
which of the following countries?
(a) Peru
(b) Chile
(c) Australia
(d) East Timor
EXPLANATION:
Valparaíso, capital of Valparaíso region in central Chile. It lies on the south side of a broad, open bay of
the Pacific Ocean, northwest of the national capital of Santiago. The city stands on the slopes of a
semicircular spur of the coastal mountain range that ends in the rocky peninsula of Point Angeles. This
point affords good shelter to the bay from southerly and westerly winds but leaves it open to those from
the north. The historic quarter of the city was designated a UNESCO World Heritage site in 2003. The El
Niño weather pattern has caused droughts and hotter-than-usual temperatures along the west of South
America this year, increasing the risk of forest fires. There were deadliest forest fires in the Valparaíso
region. So, Option (b) is correct.

SIA-A-GS I (20-A)
25. Consider the following statements :
1. Green Hydrogen is a zero-carbon fuel produced by the process of electrolysis generated from
renewable energy.
2. The SIGHT Programme, under the PM-KUSUM scheme, provides financial incentives for the
domestic manufacturing of electrolysers and the production of Green Hydrogen.
3. India’s first Green Energy Mobility Project is set up at Uttrakhand.
How many of the above statements are correct?
(a) Only one
(b) Only two
(c) All three
(d) None
EXPLANATION:
Green Hydrogen is produced using electrolysis of water with electricity generated by renewable energy. Green
hydrogen can be used as fuel for vehicles, offering a clean alternative to gasoline and diesel. Hydrogen fuel
cell vehicles emit only water and heat, making them a zero-emission transportation option. So, Statement
1 is correct.
The Strategic Interventions for Green Hydrogen Transition (SIGHT) programme is a financial measure under
the National Green Hydrogen Mission( not PM KUSUM scheme) with an outlay of ₹17,490 crores for both
domestic manufacturing of electrolysers and production of green hydrogen. So, Statement 2 is not correct.
Prime Minister laid the foundation stone of the first Green Hydrogen Mobility Project of India in Leh Ladakh
(Not in Uttarakhand) in August 2022. The first-of-its-kind Green Hydrogen Mobility Project at 11,562 ft is
co-located with a dedicated Solar plant of 1.7 MW for providing renewable power. A unique feature of this
project is that the fuel cell buses are designed for operation in sub-zero temperatures in rarefied
atmospheres, which is typical for high-altitude locations. So, Statement 3 is not correct.

26. Consider the following statements regarding battery technology :


1. Lithium-ion batteries are safer than Lithium-polymer batteries.
2. Lithium-polymer batteries have higher flexibility in shape than Lithium-ion batteries.
3. Sodium-ion batteries offer better performance and safety than both Lithium-ion and Li-polymer
batteries.
4. Sodium-ion batteries have the potential to be cheaper than Lithium-ion and Lithium-polymer
batteries.
How many of the above statements are correct?
(a) Only one
(b) Only two
(c) Only three
(d) All four
EXPLANATION:
Lithium polymer and lithium-ion batteries are generally safe to use, but there are some differences. Lithium
polymer batteries are safer as they are more stable and have small risks of leakage or explosion. On the
other hand, lithium-ion batteries have a higher risk of thermal runaway, which can occur under extreme
conditions or misuse. Thus, Lithium-polymer is even safer than lithium-ion, as there’s less risk of leaking
the electrolytic component. So, Statement 1 is not correct.
Lithium Polymer Batteries are distinct from the more commonly known lithium-ion batteries as they utilize
a solid or gel-like electrolyte as opposed to a liquid form. This differentiation in composition provides a
Lithium polymer battery with a notable edge as adaptability in shape and size. So, Statement 2 is correct.
Sodium-ion batteries (NIBs, SIBs, or Na-ion batteries) are several types of rechargeable batteries, which
use sodium ions (Na+) as its charge carriers. Both Lithium-ion and Lithium-polymer batteries have higher
performance compared to Sodium batteries Rechargeability. Sodium batteries have a shorter lifespan and

SIA-A-GS I (21-A)
can be recharged fewer times compared to Lithium batteries. Lithium batteries have a longer lifespan and
can be recharged more times compared to Sodium batteries.
Sodium-ion batteries are generally considered safer than lithium-ion batteries, as they are less prone to
overheating and catching fire. Although several experimental lithium batteries have shown incredible
resistance to damage that would make current batteries explode. So, Statement 3 is not correct.
Sodium is nearly 1000 times more abundant than lithium and is less rare a mineral. So sodium-ion
electrochemical energy storage would be a cheaper and potentially more feasible source of energy due to its
comparative abundance in the world around us. Thus, due to lower cost, higher availability, and reduced
environmental impact, of Sodium-ion batteries have a potential alternative to Lithium-ion and Lithium-
polymer batteries. So, Statement 4 is correct.

27. Consider the following :


1. Human papillomavirus
2. Hepatitis B Virus
3. Hepatitis C Virus
4. Dengue virus
How many of the above-mentioned viruses have DNA as their genetic material?
(a) Only one
(b) Only two
(c) Only three
(d) All four
EXPLANATION
Human papillomaviruses (HPVs) are small, non-enveloped, double-stranded DNA viruses that belong to
the Papovaviridae family. The circular DNA virus is responsible for causing multiple epithelial lesions and
cancers. HPV is highly transmissible.It can manifest as cutaneous and anogenital warts, which, depending
on the subtype, may progress to carcinoma. So, Statement 1 is correct.
Hepatitis B virus (HBV) is a small (3200 base pair)DNA virus belonging to the Hepadnaviridae family. It is
a partially double-stranded DNA virus with a small viral genome. Chronic HBV infection remains a global
public health problem. HBV replicates in the liver cells of humans and other higher primates and produces
an excess of the viral envelope protein (Hepatitis B surface antigen, HBsAg) that circulates in the blood. If
left untreated, chronic HBV infection can progress to end-stage liver disease, such as liver cirrhosis and
hepatocellular carcinoma (HCC). So, Statement 2 is correct.
Hepatitis C virus (HCV) is a single-stranded RNA virus in the Flaviviridae family. It causes acute hepatitis
with a high propensity for chronic infection. The genetic material of the Hepatitis C Virus (HCV) consists

SIA-A-GS I (22-A)
of single-strained RNA.Chronic HCV infection can progress to severe liver disease, including cirrhosis and
hepatocellular carcinoma. So, Statement 3 is not correct.
The dengue virus genome is a single strand of RNA. It is referred to as positive-sense RNA because it can
be directly translated into proteins. The dengue viruses are members of the genus Flavivirus in the
family Flaviviridae. Flavivirus includes yellow fever, West Nile, Japanese encephalitis, and tick-borne
encephalitis viruses. The viral genome encodes ten genes. The genome is translated as a single, long
polypeptide and then cut into ten proteins. So, Statement 4 is not correct.

28. Consider the following statements :


1. Sound is a longitudinal wave, while electromagnetic waves are transverse waves.
2. Sound is a mechanical wave, while electromagnetic waves are non-mechanical waves.
Which of the statements· given above is/are not correct?
(a) 1 Only
(b) 2 Only
(c) Both 1 and 2
(d) Neither 1 nor 2
EXPLANATION:
Sound waves in air (and any fluid medium) are longitudinal waves because particles of the medium through
which the sound is transported vibrate parallel to the direction in which the sound wave moves. Whereas,
Electromagnetic waves are transverse waves. That means the electric and magnetic fields change (oscillate)
in a plane that is perpendicular to the direction of propagation of the wave. Also, note that electric and
magnetic fields in an EM wave are also perpendicular to each other. So, Statement 1 is correct.
A sound wave is a mechanical wave that propagates along or through a medium by particle-to-particle
interaction. As a mechanical wave, it cannot travel through a vacuum and requires a medium to move from
its source to a distant location. Whereas Electromagnetic waves differ from mechanical waves in that they
do not require a medium to propagate. Electromagnetic waves are non-mechanical waves that do not need
any medium for propagation. This means that electromagnetic waves can travel not only through air and
solid materials but also through the vacuum of space. So, Statement 2 is correct.

29. Recently, the Indian Institute of Sciences and Jawaharlal Nehru Centre for Advanced Scientific
Research made a path-breaking discovery to overcome the limitation of Carnot limit, which is related
to :
(a) Efficiency of heat engines
(b) Gain in fusion energy
(c) Travelling more than the speed of light
(d) Attaining superconductivity in wire
EXPLANATION:
The Carnot limit is related to the Efficiency of heat engines. The Carnot limit defines a theoretical threshold
for the efficiency of heat engines, constraining the amount of heat that can be effectively converted into
useful work.
Recently, researchers at the Indian Institute of Sciences and Jawaharlal Nehru Centre for Advanced
Scientific Research achieved a significant breakthrough by developing a novel micro heat engine that
surpasses this limitation in laboratory conditions. Whenever engineers endeavour to innovate in heat-based
engine design, they inevitably confront this fundamental efficiency barrier.
Hence, the Carnot Limit serves as an absolute boundary on the efficiency of harnessing heat energy for
productive work. So, Option (a) is correct.

SIA-A-GS I (23-A)
30. Consider the following statements regarding the Nuclear Non-Proliferation Treaty (NPT) :
1. The International Atomic Energy Agency has the exclusive right to verify all state members of the
NPT to prevent the formation of nuclear explosive devices.
2. The Treaty on the Non-Proliferation of Nuclear Weapons cannot be amended under any
circumstances.
Which of the statements given above is/ are correct?
(a) 1 only
(b) 2 only
(c) Both 1 and 2
(d) Neither 1 nor 2
EXPLANATION:
The Nuclear Non-Proliferation Treaty was an agreement signed in 1968 by several of the major nuclear and
non-nuclear powers that pledged their cooperation in stemming the spread of nuclear technology. The
International Atomic Energy Agency (IAEA) is not a party to the Non-Proliferation Treaty (NPT) but is
entrusted with a key verification role under it. Under the NPT, the IAEA has a specific role as the
international safeguards inspectorate.
Each non-nuclear-weapon State party(not all members of the treaty) is required under Article III of the NPT
to conclude a comprehensive safeguards agreement (CSA) with the IAEA to enable the IAEA to verify the
fulfillment of their obligation under the Treaty to prevent the diversion of nuclear energy from peaceful uses
to nuclear weapons or other nuclear explosive devices. So, Statement 1 is not correct.
According to Article VIII of the Treaty on the Non-proliferation of Nuclear Weapons (NPT),
➢ Any Party to the Treaty may propose amendments to this Treaty. The text of any proposed amendment
shall be submitted to the Depositary Governments which shall circulate it to all Parties to the Treaty.
Thereupon, if requested to do so by one-third or more of the Parties to the Treaty, the Depositary
Governments shall convene a conference, to which they shall invite all the Parties to the Treaty to
consider such an amendment.
➢ Any amendment to this Treaty must be approved by a majority of the votes of all the Parties to the
Treaty, including the votes of all nuclear-weapon States Party to the Treaty and all other Parties which,
on the date the amendment is circulated, are members of the Board of Governors of the International
Atomic Energy Agency. So, Statement 2 is not correct.

31. Consider the following statements regarding the International Criminal Court (ICC) :
1. ICC can investigate individuals from non-member states if the alleged offences took place in a
member state’s territory.
2. ICC should conduct an investigation on war crimes if a crime is referred by the United Nations
Security Council.
3. The verdict of this court is final, and no appeal will be allowed.
How many of the above statements is/are correct?
(a) Only one
(b) Only two
(c) All three
(d) None
EXPLANATION:
The International Criminal Court, established in 2002, is a permanent international court established to
investigate, prosecute and try individuals accused of committing the most serious crimes of concern to the
international community as a whole, namely the crimes of genocide, crimes against humanity, war crimes
and the crime of aggression.
The ICC is based in The Hague, a city in the Netherlands that hosts many international institutions
and has field offices in several countries. The court has eighteen judges, each from a different member

SIA-A-GS I (24-A)
country and elected by the member states. It requires its members to seek a gender-balanced bench,
and the judiciary must include representatives of each of the United Nations' five regions.
The Court may exercise jurisdiction in a situation where genocide or the intent to destroy in whole or in
part a national, ethnic, racial, or religious group, crimes against humanity or war crimes, crimes of
aggression, or the use or threat of armed force were committed on or after 1 July 2002 and
➢ The prosecutor can launch an investigation into a member state proprio motu, or "on one's own
initiative."
➢ The court could investigate individuals from nonmember states if the alleged offences took place in
a member state's territory, if the nonmember state accepts the court's jurisdiction, or with the
Security Council's authorisation. So, Statement 1 is correct.
➢ A member country can refer a situation within its territory to the court;
➢ The UN Security Council can refer to a situation. So, Statement 2 is correc t.
Both the Prosecutor and the Defense have the right to appeal a Trial Chamber's decision on the verdict
(decision on the guilt or innocence of the accused) and the sentence of ICC. The victims and the convicted
person may appeal an order for reparations. An appeal is decided by five judges of the Appeals Chamber,
who are never the same judges as those who gave the original verdict. The Appeals Chamber decides
whether to uphold the appealed decision, amend it, or reverse it. This is thus the final judgment unless the
Appeals Chamber orders a re-trial before the Trial Chamber. Thus, the verdict is not final and subject to
appeal. So, Statement 3 is not correct.

32. Consider the following countries :


1. Saudi Arabia
2. Ethiopia
3. Malaysia
4. Algeria
5. Egypt
How many of the countries mentioned above are a part of the ‘BRICS-plus’ grouping?
(a) Only two
(b) Only three
(c) Only four
(d) Only five
EXPLANATION:
BRICS is an acronym for Brazil, Russia, India, China, and South Africa. The term was initially created as
BRIC (without South Africa, which joined the bloc in 2010) by Goldman Sachs economist Jim O'Neill in
2001. BRICS is home to 3.27 billion people, or 41.13 % of the world's population. With China and India
being the world's most populous countries, they have by far the largest population share (87 %) in BRICS.
➢ At the 15th BRICS summit in Johannesburg on January 1, 2024, the 5 countries Iran, Saudi Arabia,
United Arab Emirates, Egypt and Ethiopia joined the alliance. In total, BRICS Plus now comprises 10
countries that are emerging economies. Additionally, Argentina's new president, Javier Milei, has
withdrawn the country from its planned entry into the expanding BRICS. Thus, Malaysia and Algeria
are not part of the BRICS Plus grouping. So, Option (b) is correct.

SIA-A-GS I (25-A)
33. Consider the following statements with reference to the Wassenaar Arrangements :
1. All permanent members of the United Nations Security Council are members of the Wassenaar
Arrangement.
2. Its decision is binding on all members.
3. All members of this arrangement should be members of the Nuclear Suppliers Group or Chemical
Weapons Convention.
How many of the above statements is/are correct?
(a) Only one
(b) Only two
(c) All three
(d) None
EXPLANATION:
The Wassenaar Arrangement (WA) is an elite club of countries that subscribe to arms export controls,
similar to the Nuclear Suppliers Group and the Missile Technology Control Regime. The body came into
being in 1996 to succeed the Cold War-era Coordinating Committee for Multilateral Export Controls.
The name comes from Wassenaar, a suburb of the Hague, where the agreement to start such a multi-lateral
cooperation was reached in 1995. It has been established to contribute to regional and international security
and stability by promoting transparency and greater responsibility in transfers of conventional arms and
dual-use goods and technologies, thus preventing destabilising accumulations. The aim is also to prevent
the acquisition of these items by terrorists.
The Wassenaar Arrangement has 42 members, the latest entrant being India. Except for China, all the
other permanent members of the U.N. Security Council are signatories of the WA, which is headquartered
in Vienna. So, Statement 1 is not correct.
➢ To be members, states must produce and export arms or associated dual-use technologies. They also
must implement national export control laws that prohibit the sale of arms or sensitive dual-use goods
to areas of concern. Members are expected to behave in accordance with international non-proliferation
norms and standards, like the Nuclear Non-Proliferation Treaty (NPT), the Missile Technology Control
Regime (MTCR), the Chemical Weapons Convention (CWC), and the UN Register of Conventional
Arms.WA members are also expected to maintain export controls based on the control lists of the WA: the
Munitions List and the Dual-Use List.

SIA-A-GS I (26-A)
➢ This regime is a voluntary association, not bound by a treaty, and therefore has no formal mechanism
to enforce compliance. So, Statement 2 is not correct.
➢ Even though India is not a signatory to the Non-Proliferation Treaty on nuclear weapons, it has managed
to make its entry into the Wassenaar group, which would enhance its credentials in the field of non-
proliferation. India has managed to do that by updating its export control lists earlier this year to bring
it in line with international standards, including those required by the WA. So, Statement 3 is not
correct.

34. Consider the following statements with respect to the UDGAM (Unclaimed Deposits Gateway to Access
inforMation) portal :
1. It is a centralised Web Portal which facilitates registered users to search unclaimed
deposits/accounts across multiple banks in one place.
2. The user can claim his/her unclaimed deposits through the UDGAM portal or from the National
Housing Bank.
3. UDRN is a unique number generated through Core Banking Solution to settle claims received
from customers/depositors seamlessly.
How many of the above statements is/are correct?
(a) Only one
(b) Only two
(c) All three
(d) None
EXPLANATION:
UDGAM refers to Unclaimed Deposits Gateway to Access Information, which is an online portal developed
by RBI. It facilitates the registered users to search unclaimed deposits/accounts across multiple banks in
one place in a centralised manner. As of March 4, 2024, 30 banks are part of the UDGAM portal, and they
cover around 90% of unclaimed deposits (in value terms) in the Depositor Education and Awareness (DEA)
Fund of RBI. All unclaimed deposits/accounts that are part of the Depositor Education and Awareness
(DEA) Fund of RBI can be searched in the UDGAM portal.
So, Statement 1 is correct.
The UDGAM portal facilitates only the search of unclaimed deposits/accounts across multiple banks in one
place. It provides information on the claim/settlement process of each bank (which will be available in the
search result). Hence, the user can settle/ claim his/her unclaimed deposits only from the respective bank,
not through the UDGAM portal or from the National Housing Bank (NHB).
So, Statement 2 is not correct.
Unclaimed Deposit Reference Number (UDRN) is a unique number generated through Core Banking
Solution (CBS) by banks and assigned to each Unclaimed account/ deposit transferred to the Depositor
Education and Awareness (DEA) Fund of RBI. This number is used so that the account holder or the bank
branch where the account is maintained cannot be identified by any third party. The UDRN enables the
bank branches to seamlessly settle claims received from customers/depositors who have made successful
searches in the UDGAM portal. All the 30 banks on-boarded to the UDGAM portal have put in place the
requirements to generate UDRN during the development of the portal.
So, Statement 3 is correct.

35. Consider the following statements :


1. Both Mahavira and Buddha felt that only those who left their homes could gain true knowledge.
2. Both Mahavira and Buddha admitted women in their Sangha initially.
Which of the above statements is/are correct?
(a) 1 Only
(b) 2 Only
(c) Both 1 and 2
(d) Neither 1 nor 2

SIA-A-GS I (27-A)
EXPLANATION:
Both the Mahavira and the Buddha felt that only those who left their homes could gain true knowledge.
They arranged for them to stay together in the sangha, an association of those who left their homes. Men
and women who joined the sangha led simple lives. They meditated for most of the time and went to cities
and villages to beg for food during fixed hours. They taught others and helped one another. They also held
meetings to settle any quarrels that took place within the sangha. So, Statement 1 is correct.
The Sangha was the religious order of the Buddhists. It was a well-organised and powerful institution which
popularised Buddhism. Membership was open to all persons, irrespective of caste, above fifteen years of
age. Criminals, lepers and persons affected by infectious diseases were not given admission into the Sangha.
Initially Buddha was not in favour of admitting women in the Sangha. However, he admitted them at the
repeated requests of his chief disciple, Ananda and his foster mother, Mahapajapati Gotami.
Mahavira taught a simple doctrine: men and women who wished to know the truth must leave their homes.
They must follow very strictly the rules of ahimsa, which means not hurting or killing living beings. Jainism
did establish a monastic order for women. The traditional Jaina account of the growth of the sangha during
the lifetime of Mahavira, in fact, gives greater prominence to women. According to the Kalpa Sutra, when
Mahavira died, there were 14,000 monks and 36,000 nuns, 159,000 laymen, and 318,000 laywomen.
Thus, Mahavira initially admitted women, whereas Buddha did not admit women initially. So, Statement
2 is not correct.

36. ‘Ol Chiki’ script is used to write which of the following scheduled languages of India ?
(a) Konkani
(b) Bengali
(c) Santhali
(d) Bodo
EXPLANATION:
According to the 2011 Census of India, there are over 70 lakh (seven million) people who speak Santali
across the country, and the Santhal community is the third-largest tribe in India, concentrated in seven
states in large numbers, including in West Bengal, Odisha and Jharkhand. But their geographic
distribution is not limited to India; the community is also spread across Bangladesh, Bhutan and Nepal.
Santhals speak Santhali, which has a script called ‘Ol chiki’ invented by Pundit Raghunath Murmu. Ol-
Chiki is a medium of instruction in many schools across the State. Even Santhal is a subject in post-
graduate class.
In 2003, the 92nd Constitutional Amendment Act added Santali to Schedule VIII to the Constitution of
India, which lists the official languages of India, along with the Bodo, Dogri and Maithili languages. This
addition meant that the Indian government was obligated to undertake the development of the Santali
language and to allow students appearing for school-level examinations and entrance examinations for
public service jobs to use the language.
Thus, the ‘Olchiki’ script is used to write the Santhali language of India. So, Option (c) is correct.

37. With reference to the Post-Mauryan period (200 BCE – 300 AD), consider the following statements :
1. The most significant development of this period in the economic sphere was an increase in the
volume of international trade.
2. Buddhism and its monasteries played an important role in facilitating international trade.
Which of the statements given above is/are correct?
(a) 1 only
(b) 2 only
(c) Both 1 and 2
(d) Neither 1 nor 2
EXPLANATION:
The post-Mauryan period between c.200 BCE to 200 CE To know about this era variety of sources are
available, including both foreign as well as indigenous sources. The most important is the Periplus of the
Erythraean Sea, which gives rich data on sea trade between India and Rome. Some aspects of this trade
SIA-A-GS I (28-A)
can be understood from works by Classical writers like Pliny and Tiberius. The most significant development
of this period in the economic sphere was an increase in the volume of international trade. India had trade
links with the Western world as well as Southeast Asia. In the Western world, most of the trade was with
the Roman Empire. This is attested by both classical sources as well as archaeological data. Trade was
conducted through both land and sea routes. The sea route was possibly more important. Trade started
from the ports in Egypt and ended at the ports located on the eastern coast of India. So, Statement 1 is
correct.
According to D.D. Kosambi, the monasteries were important consumers of items like cloth and materials
required in rituals; hence, they created a demand for these goods. At times, if required, they could also
provide merchants with the required capital or loans. It is important to note that the monasteries were
located on important trade routes. Some archaeological finds even indicate the involvement of monks in
trade. In one monastery at Devnimori in Gujarat, Roman amphorae sherds with traces of wine were
discovered. A monastery located in Pushkalavati even had a room full of distillation apparatus. Hence, It is
known that Buddhism and its monasteries played an important role in facilitating trade. So, Statement 2
is correct.

38. In the context of India's cultural history, "The Dying Princess", one of the greatest paintings, has been
depicted in which of the following caves ?
(a) Barabar Cave
(b) Ellora Cave
(c) Ajantha Cave
(d) Ratnagiri Cave
EXPLANATION:
'The Dying Princess' is a painting at Ajanta that represents the last moments of Sundari, the heartbroken
wife of Buddha's brother, Nanda, who left her to become a monk. Emotion and pathos are expressed here
by the controlled turn and poise of the body and the eloquent gestures of hands and eyes. The dying princess
is reclining against a bolster, and a female attendant is holding her. The bent head of the princess, her
lowered eyes, the drooping limbs and the sad, tense female attendants all hint at imminent death. Though
deserted by her husband, in her last moments, the dying princess is surrounded by deeply concerned
attendants. One is fanning her while clutching her own heart in grief. In an attached chamber stand two
other attendants, one with a Persian cap and another with a distinctly Afro-Asian face signalling someone
to bring something. Her eyes are wide with fear. Yet another is checking the princess' pulse, sensing death
inch closer. So, Option (c) is correct.

39. Consider the following statements:


1. In 1606, the Dutch established their first factory at the port of Masulipatnam.
2. In 1721, the French acquired Mahe on the Malabar Coast and established Fort Louis.

SIA-A-GS I (29-A)
Which of the above statements is/are correct ?
(a) 1 only
(b) 2 only
(c) Both 1 and 2
(d) Neither 1 nor 2
EXPLANATION:
In 1602, the United East India Company of the Netherlands was formed and permitted by the Dutch
government to trade in the East Indies, including India. The company was granted a 21-year charter with
rights to trade exclusively in Asia and to buy valuable spices, such as nutmeg, mace, and cloves.
After they arrived in India, the Dutch East India Company established its first factory at Petapuli on the
North Coromandel Coast in 1606. Another factory was established in the same year at the port of
Masulipatnam. In 1610, a factory at Pulicat was established, which also became the headquarters of the
Dutch directorate of the Coromandel. Fort Geldria was constructed at Pulicat in 1613. The Dutch established
factories on the Coromandel coast and in Gujarat, Uttar Pradesh, Bengal, and Bihar. So, Statement 1 is
not correct.
French under the reign of Louis XIV, the king’s famous minister Colbert laid the foundation of the Compagnie
des Indes Orientales (French East India Company) in 1664. The Compagnie des Indes Orientales was granted
a 50-year monopoly on French trade in the Indian and Pacific Oceans. The French were the last Europeans
to come to India with the purpose of trade.
The French established their first factory in India at Surat in 1668. When the British established its Empire
on the west coast, The French were inspired to establish their dominion in Mahe. Their first location in
Kerala was Thalassery. The French Company acquired Mahe on the Malabar Coast in 1721, followed by
Yanam in 1731 and Karaikal in 1758. The headquarters of the French were in Pondichery, where Francois
Martin established Fort Louis during 1700-1707. So, Statement 2 is not correct.

40. Consider the following pairs :


S. No Swadeshi Authors
articles or books
1. Swadeshi Bhandar - Sarala Devi
2. Lakshmi Bhandar - Rabindranath Tagore
3. Indian stores - Bal Gangadhar Tilak
How many of the above pairs are correctly matched?
(a) Only one
(b) Only two
(c) All three
(d) None
EXPLANATION:
Writer and nationalist Rabindranath Tagore started the ‘Bhandar’ - Swadeshi Periodical in 1897 from
Kolkata. Bhandar was well known for its writings and editorial pieces on the freedom movement and the
concept of revivalism. The journal was rife with essays on the spirit of self-help and nationalism. The journal
regularly invites topical questions posed by it from its readers. So, Pair (1) is not correct.
Sarla Devi Chaudhurani was a prominent freedom fighter from Bengal as well as Punjab. Sarla Devi worked
as the editor of the Bengali Journal Bharti. She was a niece of Rabindranath Tagore and had connections
with national leaders such as Satyendra Nath Tagore and Chaupekar brothers. She opened Laksmi Bhandar
to make popular Swadeshi goods.
She became an active member of Bharat Stree Mahamandal of Lahore, which was meant to organise women
of all colours and creeds for the common cause and moral and material development of the women of the
nation. So, Pair (2) is not correct.
Jogesh Chandra, writer and author, was educated in Calcutta and Oxford. Jogesh Chandra came in close
touch with the great Surendranath Banerjee, ever to be remembered as the ‘Father of Indian Nationalism’.
His works include Indian Stores, published in 1901. So, Pair (3) is not correct.

SIA-A-GS I (30-A)
41. Arrange the Social Reform Movements given below in the correct chronological order:
1. Tattwa Bodhini Sabha
2. Prarthana Samaj
3. Deva Samaj
4. Satya Shodhak Samaj
Select the correct code from the options given below:
(a) 1-2-3-4
(b) 1-2-4-3
(c) 3-4-1-2
(d) 3-2-4-1
EXPLANATION:
➢ Debendranath Tagore established Tattvabodhini Sabha on 6 October 1839 at Jorasanko, Calcutta (now
Kolkata). Debendranath created the Sabha to spread the knowledge and the truth of the Shastras and
the Vedantas among the country's people. The objectives of the Sabha were to prevent cultural intrusion
from the West and protect the educated youths from being influenced by the Christian missionaries. The
Sabha helped increase social consciousness in society by trying to eliminate religious dogmas. The Sabha
continuously worked for public welfare and helped with people's political awakening against the British
government.
➢ Keshab Chandra Sen helped Atmaram Pandurang found the Prarthana Samaj in 1867 in Bombay. A
precursor of the Prarthana Samaj was the Paramahansa Sabha, something like a secret society to spread
liberal ideas and encourage the breakdown of caste and communal barriers. Mahadeo Govind Ranade
(1842–1901) joined the samaj in 1870, and much of the popularity of and work done by the society was
due to his efforts. His efforts made the Samaj gain an all-India character. Other leaders of the Samaj
were R.G. Bhandarkar (1837- 1925) and N.G. Chandavarkar (1855–1923). The emphasis was on
monotheism, but on the whole, the samaj was more concerned with social reforms than with religion.
The Prarthana Sabha was very attached to the bhakti cult of Maharashtra. The samaj relied on education
and persuasion and not on confrontation with Hindu orthodoxy.
➢ Dev Samaj is a religious and social reform society founded in February 1887 in Lahore by Pandit Shiv
Narayan Agnihotri. The society emphasised the eternity of the soul, the supremacy of the guru, and the
need for good action. It called for ideal social behaviour, such as not accepting bribes, avoiding
intoxicants and non-vegetarian food, and keeping away from violent actions. Its teachings were compiled
in a book, Deva Shastra. Agnihotri spoke against child marriage.
➢ Jyotirao or Jyotiba Phule founded the Satyashodhak Samaj (Truth Seekers' Society) in 1873, with the
leadership of the samaj coming from the backward classes, malis, telis, kunbis, saris, and dhangars.
The main aims of the movement were (i) social service and (ii) the spread of education among women and
lower caste people. Phule's principal objective was to alleviate the conditions of the depressed classes in
Maharashtra and to prevent their economic and social exploitation by the oppressive nexus of
moneylenders and the colonial administration.
Therefore, the correct chronological order of the given social reform movement is 1-2-4-3. So, Option (b) is
correct.

42. Consider the following statements :


1. Mahatma Gandhi declared that if this programme were implemented completely, swaraj would be
ushered in within a year.
2. Some leaders like Muhammad Ali Jinnah, Annie Besant, G.S. Kharpade, and B.C. Pal left the
Congress during this programme.
3. During this movement, Jamia Millia at Aligarh, Kashi Vidyapeeth, Gujarat Vidyapeeth, and Bihar
Vidyapeeth were established.
4. The Congress volunteer corps emerged as the parallel police force during this phase.
SIA-A-GS I (31-A)
The above statements refer to which movement in the Indian freedom struggle?
(a) Rowlatt Satyagraha
(b) Salt Satyagraha
(c) Khilafat movement
(d) Individual Satyagraha
EXPLANATION:
The Khilafat Movement (1919-1920) was a movement of Indian Muslims, led by Muhammad Ali and
Shaukat Ali, in protest against British policies towards Turkey and the dismantling of the Ottoman Empire
after World War I. The Congress supported the movement and Mahatma Gandhi sought to conjoin it to the
Non-cooperation Movement. During the movement, the participants were supposed to work for Hindu-
Muslim unity and removal of untouchability, all the time remaining non-violent.
➢ At a special session in Calcutta in September 1920, the Congress approved a non-cooperation
programme till the Punjab (Jallianwala Bagh Massacre) and Khilafat wrongs were removed and swaraj
was established.
➢ In December 1920, at the Nagpur session of the Indian National Congress, Gandhi declared that if the
non-cooperation programme were implemented completely, swaraj would be ushered in within a year.
Many groups of revolutionary terrorists, especially those from Bengal, also pledged support to the
Congress programme.
➢ At this stage, some leaders like Muhammad Ali Jinnah, Annie Besant, G.S. Kharpade, and B.C. Pal left
Congress as they believed in a constitutional and lawful struggle. In contrast, some others, like
Surendranath Banerjee, founded the Indian National Liberal Federation and played a minor role in
national politics henceforward.
➢ During the Khilafat movement, educational institutions were organised under the leadership of Acharya
Narendra Dev, C.R. Das, Lala Lajpat Rai, Zakir Hussain, and Subhash Bose (who became the principal
of National College at Calcutta). They included Jamia Millia at Aligarh, Kashi Vidyapeeth, Gujarat
Vidyapeeth, and Bihar Vidyapeeth. The Congress volunteer corps emerged as the parallel police during
this phase. So, Option (c) is correct.

43. Consider the following pairs:


S.No Personality Contribution to Freedom Struggle
1. Kamaladevi - Unfurled Indian flag in Stuttgart, Germany
Chattopadhyay
2. Bhikaji Cama - Lead the salt satyagraha in Bombay.
3. Vijayalakshmi - Transported bombs and weapons to
Pandit revolutionary groups in Bengal
4. Kalpana Dutt - Demanded political suffrage for women
How many above pairs are correctly matched?
(a) Only two
(b) Only three
(c) All four
(d) None
EXPLANATION:
Kamaladevi Chattopadhyay (1903 –1988) was a significant figure in the Indian National movement and
the history of women's activism across many borders. She joined the Indian National Congress in 1927
and was elected to the All-India Congress Committee within a year. During the Salt March to Dandi, she
convinced Gandhi to give women equal opportunity to be at the forefront of the March. On April 6, 1930,
a group of women led by Kamaladevi Chattopadhyaya marched towards Chowpatty, Mumbai and started
making salt on chulhas (makeshift stoves). Later, she joined Seva Dal and trained women activists.
Kamaladevi Chattopadhyay did not hoist the Indian flag in Stuttgart, Germany. So, Pair (1) is not correct.
Bhikaiji Rustom Cama, or simply Madam Cama, was one of the prominent figures in the Indian
independence movement. She was born in Bombay in a large, affluent Parsi Zoroastrian family.

SIA-A-GS I (32-A)
Madam Bhikaji Cama became the first person to hoist the Indian flag on foreign soil at the International
Socialist Conference in Stuttgart, Germany, on August 22 1907. Appealing for human rights, equality and
autonomy from Great Britain, she described the devastating effects of a famine that had struck the Indian
subcontinent. Madam Bhikaji Cama does not Lead the salt satyagraha in Bombay. So, Pair (2) is not
correct.
Vijaya Lakshmi Pandit is known for being the first woman president of the United Nations General
Assembly and headed the eighth session of the General Assembly in 1953. Her first political stint was at
the All-India Women's Conference (AIWC) in the 1930s, where she worked on bringing resolutions covering
gender rights and challenging Hindu personal laws. Vijaya Lakshmi Pandit is not associated with
Transporting bombs and weapons to revolutionary groups in Bengal. So, Pair (3) is not correct.
Kalpana Dutt was a remarkable Indian revolutionary who participated in the Chittagong Armory Raid led
by Surya Sen on April 18 1930. She studied at Calcutta University and became politically active. Kalpana
and her friend Pritilata Waddedar joined Surya Sen's Indian Republican Army while they were in college,
inspired by the story of Rani Lakshmibai of Jhansi. Kalpana Dutt was not associated with the demand for
political suffrage for women. So, Pair (4) is not correct.

44. The terms' Stack, stump and blowhole' are related to :


(a) Coastal region
(b) Glacial region
(c) Desert region
(d) Volcanic region
EXPLANATION:
Erosion along rocky coasts occurs at various rates and is dependent both on the rock type and on the
wave energy at a particular site. As a result of the conditions mentioned above, wave-cut platforms may
be incomplete, with erosional remnants on the horizontal wave-cut surface. Such resistant masses of
rock, originally parts of a cliff or hill, are called 'sea stacks', and they provide a spectacular type of
coastal landform.

A stump is the eroded remains of a sea stack, often looking like a lump of rock sticking up from its
surroundings. It is the last stage in the process that starts with a weakness in a cliff that develops into a
cave, then an arch, a stack, and finally, a stump.

SIA-A-GS I (33-A)
When sea caves grow towards the land and upwards, creating a vertical shaft that is exposed on the
surface, it results in a blowhole. Water often gushes out at the top part of the landform when waves move
to the sea cave with significant force. A blowhole is characterised by an opening on the ground and a
connection to an opening which interacts with the sea, mostly a cave.
Hence, All there are landforms in coastal regions. So, Option (a) is correct.

45. Consider the following statements with respect to the Cool temperate Western Margin Climate:
1. They are under the permanent influence of Westerlies throughout the year.
2. Throughout the year, rainfall with a slight winter maximum due to frontal cyclones.
3. It experiences moderately warm summers and mild winters because of the Warm North Atlantic
Drift.
How many of the statements given above is/are correct?
(a) Only one
(b) Only two
(c) All three
(d) None

SIA-A-GS I (34-A)
EXPLANATION:
The cool, temperate western margin climate is under the permanent influence of the Westerlies all around
the year. They are also regions of much cyclonic activity, typical of Britain, and are thus said to experience
the British type of climate.
From Britain, the climatic belt stretches far inland into the lowlands of North-West Europe, including
regions such as northern and western France, Belgium, the Netherlands, Denmark, western Norway and
also north-western Iberia. There is so much oceanic influence on both the temperature and the
precipitation that the climate is also referred to as the North-West European Maritime Climate.
So, Statement 1 is correct.
The British type of climate has adequate rainfall throughout the year with a tendency towards a slight
winter or autumn maximum from cyclonic sources (due to frontal cyclones). Since the rain-bearing winds
come from the west, the western margins have the heaviest rainfall. The amount decreases eastwards with
increasing distance from the sea. So, Statement 2 is correct.
The mean annual temperatures are usually between 40°F and 60°F. Summers are, in fact, never very
warm. Monthly temperatures of over 65°F, even in mid-summer, are rare. Winters are abnormally mild,
and no places record mean January temperatures below freezing-point in north-western Europe. This is
attributable to the warming effect of the warm North Atlantic Drift and the prevalence of the South-
Westerlies.
Night frosts do occur, and snow falls in winter, too. Sometimes, unusual cold spells caused by the invasion
of cold polar continental air from the interiors may hit the western margins for a number of weeks. The
climate of the maritime regions as a whole may be described as equable with moderately warm summers
and fairly mild winters. So, Statement 3 is correct.

46. Consider the following statements with respect to temperature inversion:


1. It is a phenomenon where the temperature increases with increasing altitude in the troposphere.
2. It is most commonly observed during long winter nights with clear skies and calm winds.
3. It can trap pollutants closer to the ground, leading to poor air quality.
Which of the statements given above is/are correct ?
(a) 1 only
(b) 2 only
(c) 2 and 3 only
(d) 1, 2 and 3
SIA-A-GS I (35-A)
EXPLANATION:
In general conditions, the temperature decreases with height in the troposphere. But sometimes the
temperature increases with height (increasing altitude) under special circumstances is called inversion of
temperature. In simple words, a Temperature inversion is a phenomenon where the temperature increases
with increasing altitude in the troposphere. It occurs when a layer of warm air traps cool air near the Earth's
surface or in the upper atmosphere.
Temperature inversion near the surface may occur under such conditions as long and clear winter nights,
clear skies, calm air, snow-covered surfaces etc. Over polar areas, temperature inversion is normal
throughout the year. The heat of the day is radiated off during the night, and by early morning hours, the
earth is cooler than the air above. So, Statements 1 and 2 are correct.
Surface inversion promotes stability in the lower layers of the atmosphere. Smoke and dust particles get
collected beneath the inversion layer and spread horizontally to fill the lower strata of the atmosphere. Dense
fogs in the mornings are common occurrences, especially during the winter season. This inversion commonly
lasts for a few hours until the sun comes up and begins to warm the earth.
Surface temperature inversions play a major role in air quality, especially during the winter when these
inversions are the strongest. Pollutants from vehicles, wood burning, area sources, and industry become
trapped near the ground during inversions, leading to poor air quality. Thus, temperature inversion can trap
pollutants closer to the ground, leading to poor air quality. So, Statement 3 is correct.

47. Match the following ocean currents in List I with their locations in List II :
S. No List I List II
1. Kuroshio - The eastern coast of
Current North America
2. Labrador - The western coast of
Current North America
3. Humboldt - The western coast of
Current South America
4. Agulhas - The western coast of
Current Africa
How many of the above pairs is/are correctly matched ?
(a) Only one
(b) Only two
(c) Only three
(d) All four
EXPLANATION:
Kuroshio is a strong surface oceanic current of the Pacific Ocean. It is found off the eastern coast of Japan.
It is the northeasterly flowing continuation of the Pacific North Equatorial Current between Luzon of the
Philippines and the east coast of Japan.
SIA-A-GS I (36-A)
The Kuroshio exhibits distinct seasonal fluctuations. It is strongest from May to August. Receding some in
late summer and autumn, it begins to increase from January to February only to weaken in early spring.
Similar to the Gulf Stream (Atlantic) in its creation and flow patterns, the Kuroshio has an important
warming effect upon the south and southeast coastal regions of Japan as far north as Tokyo. So, Pair (1)
is not correct.

The Labrador Current is found off the east coast of Canada in the North Atlantic Ocean. Thus, it is found
on the north-eastern coast of North America (not on the Western coast of North America).
It is a surface oceanic current in the North Atlantic Ocean that originates at the Davis Strait, where it begins
as a continuation of the West Greenland Current and the Baffin Island Current. From there, it flows
southwards along the western side of the Labrador Sea. The current passes along the coasts of Labrador,
Newfoundland, and the east coast of Nova Scotia in Canada. The Labrador Current is a cold current with
low salinity and reaches a maximum depth of around 2,000 feet.
So, Pair (2) is not correct.

The Humboldt Current (also known as Peruvian Current) is a cold water ocean current that flows north
from Antarctica along the west coast of South America, bringing nutrient-rich water to the Galapagos
Islands and helping to sustain the island's rich biodiversity. The current is driven by strong winds, which
displace the warm and nutrient-poor surface water, allowing the cold Antarctic waters to rise to the surface,
creating a phenomenon known as upwelling. So, Pair (3) is correct.

SIA-A-GS I (37-A)
Agulhas Current is a warm water current that runs south along the east coast of southern Africa from 25°S
to 40°S and is the Western Boundary Current of the South Indian Ocean. A small part of Agulhas water
apparently continues westward around the Cape of Good Hope into the Atlantic Ocean. The Mozambique
Current, between Madagascar and Africa, also feeds the Agulhas Current. So, Pair (4) is not correct.

48. Consider the following:


1. Jabel Ali of UAE
2. Al Haditha of Saudi Arabia
3. Eilat of Jordan
4. Haifa of Israel
5. Piraeus of Greece
How many of the above cities are part of the proposed India-Middle East-Europe-Economic Corridor
announced on the sidelines of the G20 meeting in New Delhi?
(a) Only two
(b) Only three
(c) Only four
(d) All five
SIA-A-GS I (38-A)
EXPLANATION:
India-Middle East-Europe-Economic Corridor (IMEC) will consist of railroad, ship-to-rail networks (road
and sea) and road transport routes (and networks) extending across two corridors, that is, the east corridor
connecting India to the Gulf, and the northern corridor – connecting the Gulf to Europe. As per the MoU,
the railway, upon completion, would provide a reliable and cost-effective cross-border ship-to-rail transit
network to supplement existing maritime and road transports routes.
IMEC, which promises shorter routes, is a breakthrough. It links major ports of western India including
JNPT, Kochi, Kandla and Mundra with major shipping ports of the Gulf, including Jebel Ali (UAE), Fujairah
(UAE), Ras Al-Khair (Saudi Arabia), Dammam (Saudi Arabia), Duqm (Oman), and Salalah (Oman). From
these ports, cargo will be transported by the Saudi rail network on their north-south line to the port of
Haifa in Israel through Jordan.
Haifa, being a deep seaport, can handle bulk container trains and post Panamax ships which after
transhipment, carry cargo to European ports like Piraeus, Kavala (Greece), Trieste, La Spezia (Italy),
Marseille-Fos (France), Barcelona, and Valencia (Spain). Road container trailers or container cargo trains
will thereafter transport goods across Europe. So, Statements 1 and 5 are correct.

Connectivity to Gulf ports will be operationalised by the under-construction rail network of the Gulf
Cooperation Council to connect Kuwait with Muscat on a north-south coastal rail system. Although this
will connect the gulf ports, exit across Saudi Arabia will only be possible on the Saudi Arabian Rail (SAR)
network connected to Saudi ports and operational to the borders of Jordan by a robust heavy-haul railway
system. As the SAR network ends on the Jordan border, the missing link from the last SAR railhead at Al
Haditha to Haifa becomes the most crucial challenge of IMEC.
From the Saudi border terminal of Al Haditha to Amman of Jordan, rail connectivity is already being
planned. Hence, Eilat is not part of IMEC. So, Statements 2 and 4 are correct, and Statement 3 is not
correct.

49. With reference to the PM – SURAJ portal, which of the following statements is correct?
(a) It is a single-window grievance redressal portal for Micro, Small & Medium Enterprises launched
by the Ministry of MSMEs.
(b) It is a one-stop point to provide loan assistance to eligible persons across the country,
including sanitation workers.
(c) It is a portal that provides standardised terminologies & morbidity codes for Ayurveda, Siddha
and Unani systems of medicines.
(d) It is a standard MOOC platform with course material such as filmed lectures and e-study material
in the field of social defence.
SIA-A-GS I (39-A)
EXPLANATION:
The Union Ministry of MSME has launched the CHAMPIONS portal, a Technology Control Room-Cum-
Management Information System. The system utilising modern ICT tools is aimed at assisting Indian
MSMEs march into the big league as National and Global CHAMPIONS. The CHAMPIONS stands here for
Creation and Harmonious Application of Modern Processes for Increasing Output and National Strength.
As the name suggests, the portal is basically for making the smaller units big by solving their grievances,
encouraging, supporting, helping and handholding. It is a real one-stop-shop solution for the MSME’s.
So, Option (a) is not correct.
The ‘Pradhan Mantri Samajik Utthan Evam Rozgar Aadharit Jankalyan (PM-SURAJ) scheme and its portal
The scheme has been launched to economically empower people belonging to socially backward classes,
scheduled castes, tribes and other disadvantaged sections of the society. The nationwide PM-SURAJ
national portal for credit support to disadvantaged sections and to prioritise the underprivileged
(vanchiton ko variyata). The credit support will be provided to the eligible persons across the country, and
facilitated through banks, NBFC-mfis, and other organisations. So, Option (b) is correct.
The Ministry of Ayush has developed the Code for Ayurveda, Siddha, and Unani Medicine through the
National Ayush Morbidity and Standardized Electronic Portal (NAMSTE). It is a comprehensive web portal
for Standardized Terminologies and National Morbidity Codes of Ayurveda, Siddha and Unani Systems of
Medicine and WHO-ICD-10 and ICD-11. So, Option (c) is not correct.
The Ministry for Social Justice launched an online portal, TAPAS (Training for Augmenting Productivity
and Services). TAPAS is the initiative of the National Institute of Social Defence (NISD) to provide access
to lectures by subject experts, study material and more, but in a manner that supplements the physical
classroom without compromising on the quality of teaching. The main objective of introducing the course
modules is to impart training and enhance the knowledge and skills for the capacity building of the
participants. As a standard MOOC (Massive Open Online Course) platform, TAPAS (Training for
Augmenting Productivity and Services) offers various courses in the field of social defence for the capacity
building of stakeholders. So, Option (d) is not correct.

50. Consider the following statements regarding the Scheme for the Protection and Preservation of
Endangered Languages of India :
1. The Central Institute of Indian Languages (CIIL), Mysore, seeks to protect, preserve, and document
all endangered languages.
2. In India, more than 40 languages are classified as critically endangered languages.
3. None of the Indian languages has become extinct since independence.
How many of the above statements is/are correct?
(a) Only one
(b) Only two
(c) All three
(d) None
EXPLANATION:
The Government of India has initiated a Scheme known as "Protection and Preservation of Endangered
Languages of India". Under this Scheme, the Central Institute of Indian Languages (CIIL), Mysore, works
on the protection, preservation and documentation of all the mother tongues/languages of India spoken
by less than 10,000 people, which are called endangered languages. Under this scheme, 117 endangered
languages/mother tongues have been chosen from all over India for study and documentation on a priority
basis.So, Statement 1 is correct.
➢ According to a report of the census directorate, there are 22 scheduled languages and 100 non-
scheduled languages in the country, which are spoken by a large number of people - one lakh or more.
➢ UNESCO has categorized languages on the basis of endangerment as follows:-
• Vulnerable
• Definitely Endangered
• Severely Endangered
SIA-A-GS I (40-A)
• Critically Endangered
➢ A list prepared by UNESCO has also mentioned the 42 languages or dialects in India which are
Critically endangered and they may be heading towards extinction. Critically endangered languages
are those whose youngest speakers are grandparents or older, and they speak the language partially
and infrequently. So, Statement 2 is correct.
➢ The languages or dialects which were considered endangered, include 11 from Andaman and Nicobar
Islands (Great Andamanese, Jarawa, Lamongse, Luro, Muot, Onge, Pu, Sanenyo, Sentilese, Shompen
and Takahanyilang), seven from Manipur (Aimol, Aka, Koiren, Lamgang, Langrong, Purum and Tarao)
and four from Himachal Pradesh (Baghati, Handuri, Pangvali and Sirmaudi), three from Odisha
(Manda, Parji and Pengo), two from Karnataka (Koraga and Kuruba), two from Andhra Pradesh
(Gadaba and Naiki), two from Tamil Nadu (Kota and Toda), two from Arunachal Pradesh (Mra and Na),
two from Assam (Tai Nora and Tai Rong), One from Uttarakhand (Bangani), one from Jharkhand
(Birhor), one from Maharashtra (Nihali), one from Meghalaya (Ruga) and one from West Bengal (Toto).
The UNESCO Atlas lists only five Indian languages as having been extinct since 1950. The extinct
languages are Ahom, Andro, Rangkas, Sengmai, and Tolcha, which are all languages spoken in the
Himalayan belt. Since 1971, the Census has been releasing data only on languages spoken by 10,000
people and above.
So, Statement 3 is not correct.

51. Gandhiji used the phrase Himalayan Blunder' to refer to the violence during :
(a) Rowlatt Satyagraha
(b) Non-Cooperation Movement
(c) Salt Satyagraha
(d) Quit India Movement
EXPLANATION:
Mahatma Gandhi formed a Satyagraha Sabha on February 24 1919, in Bombay to protest against the
Rowlatt Bills. He inaugurated his Satyagraha by calling upon the countrymen to observe a day of ‘hartal’
where business should be suspended, and people should fast and pray as a protest against the Rowlatt
Act. Mahatma Gandhi was arrested on his way to Delhi and Punjab to promote the Satyagraha agitation.
The news of Mahatma Gandhi’s arrest precipitated the crisis. The Punjab region as a whole and Amritsar,
in particular, witnessed the worst scenes of violence.
The government called the army and handed over the city to General Dyer, who ordered a ban on public
meetings and assemblies. On April 13, on Baisakhi, a large crowd of people gathered at Jallianwala Bagh
to attend a public meeting. Dyer, who wanted to strike terror in Punjab, ordered his troops to open fire,
killing about 1,000 and injuring more. It led to unprecedented violence across the country, as people,
infuriated by government action, took to the streets to act violently against the government.
On April 18 1919, Mahatma Gandhi decided to call off the Rowlatt Satyagraha because of the widespread
violence. He confessed publically that he committed a ‘Himalayan blunder’ by offering civil disobedience
to people who were insufficiently prepared for the discipline of Satyagraha.
So, Option (a) is correct.

52. Consider the following :


1. Perfluorocarbons
2. Nitrogen dioxide
3. Sulfur hexafluoride
4. Carbon monoxide
5. Nitrogen trifluoride
How many of the above atmospheric gases are considered Greenhouse gases?
(a) Only two
(b) Only three
(c) Only four
(d) All five
SIA-A-GS I (41-A)
EXPLANATION:
Perfluorocarbons (PFCs) are potent greenhouse gases with global warming potentials up to several
thousand times greater than CO2 on a 100-year time horizon. The lack of any significant sinks for PFCs
means that they have long atmospheric lifetimes of the order of thousands of years. Anthropogenic
production is thought to be the only source for most PFCs. So, Statement 1 is correct.
Neither nitric oxide nor nitrogen dioxide are greenhouse gases, although they are important in the process
of creation of tropospheric ozone which is a greenhouse gas. There are several sources of nitrous oxide,
both natural and anthropogenic (human), in the atmosphere with many of these sources difficult to
measure. Because of this, there is general agreement that the atmospheric sources and sinks of nitrous
oxide are difficult to bring into balance. So, Statement 2 is not correct.
Sulphur hexafluoride, also known as SF6, is a 'greenhouse gas' that has long played a part in global
warming, similar to that of carbon dioxide (CO2). SF6 is a synthetic, odourless gas that's used in the
electricity industry to keep networks running safely and reliably. It's highly stable, non-toxic, non-
flammable and electronegative, which means it will not form other compounds that will alter its state and
effectiveness. So, Statement 3 is correct.
Carbon monoxide does not cause climate change directly, its presence affects the abundance of
greenhouse gases such as methane and carbon dioxide. Carbon monoxide reacts with hydroxyl (OH)
radicals in the atmosphere, reducing their abundance. As OH radicals help to reduce the lifetimes of
strong greenhouse gases, like methane, carbon monoxide which indirectly increases the global warming
potential of these gases. Carbon monoxide in the atmosphere can also lead to the formation of the
tropospheric greenhouse gas 'ozone'. So, Statement 4 is not correct.
Nitrogen trifluoride (NF3) is an inorganic nitrogen-fluorine compound that acts as a replacement for
PFCs, specifically hexafluoroethane (C 2 F6 ). It is used most frequently in the electronics industry during
various processes including plasma etching, cleaning chambers in which silicon chips are made, and
semi-conductor and LCD panel manufacture. Additionally, it has several important application s
within the photovoltaic and chemical laser industries. NF3 is very effective in absorbing the infrared
radiation that the Earth emits. By trapping this infrared radiation, NF3 becomes a potent greenhouse gas.
So, Statement 5 is correct.

53. With reference to the National Green Tribunal, consider the following statements :
1. It is a specialised judicial body established by executive resolution by the cabinet.
2. Its orders are binding, and it has the authority to provide compensation and damages to those
affected.
3. It has jurisdiction over civil and criminal cases concerning environmental issues.
How many of the above statements is/are correct?
(a) Only one
(b) Only two
(c) All three
(d) None
EXPLANATION:
The National Green Tribunal, established in 2010, as per the National Green Tribunal Act, 2010 (not
through executive resolution by the cabinet) is a specialised judicial body equipped with expertise solely
for the purpose of adjudicating environmental cases in the country. So, Statement 1 is not correct.
The Tribunal is tasked with providing an effective and expeditious remedy in cases relating to
environmental protection, cons
ervation of forests and other natural resources and enforcement of any legal right relating to the
environment. The Tribunal's orders are binding, and it has the power to grant relief in the form of
compensation and damages to affected persons. So, Statement 2 is correct.
The Tribunal has jurisdiction over all civil cases (not criminal ) involving a substantial question relating
to the environment and the question. Additionally, any person aggrieved by an order/direction of any of
the Appellate Authorities under the legislations mentioned in Schedule I of the National Green Tribunal
SIA-A-GS I (42-A)
Act, 2010 can also challenge them before the National Green Tribunal. The Tribunal's orders are
enforceable as the powers vested are the same as in a civil court under the Code of Civil Procedure, 1908.
The Tribunal shall not be bound by the procedure laid down under the Code of Civil Procedure, 1908,
but shall be guided by principles of natural justice.
So, Statement 3 is not correct.

54. Consider the following processes:


1. Oxidation of food leading to the release of Carbon dioxide and water
2. Conversion of light energy into chemical energy and its storage
3. An endothermic reaction through which food is synthesised
How many of the above processes are involved in the respiration of living organism?
(a) Only one
(b) Only two
(c) All three
(d) None
EXPLANATION:
Respiration is the process by which fats and protein’s energy stored in organic molecules is released by
oxidation. This energy is thus made available to the living cells in the form of ATP (Adenosine Tri-
Phosphate). The O2 required for respiration is obtained from the atmosphere. ATP is the energy currency
of the cell.
Respiration is the stepwise oxidation of complex organic molecules and release of energy as ATP for various
cellular metabolic activities. It involves exchange of gases between the organism and the external
environment. The green as well as non-green plants obtain oxygen from their environment and return
carbon dioxide and water vapour into it. This mere exchange of gases is known as external respiration or
breathing in case of animals. It is a physical process. The biochemical process, which occurs within cells
and oxidises food to obtain energy, is known as cellular respiration. Therefore, Oxidation of food leading
to the release of Carbon dioxide and water is involved in the respiration of living species.
So, Statement 1 is correct.
In the process of photosynthesis, green plants and cyanobacteria can prepare their own food by trapping
light energy and converting it into chemical energy that is stored in the bonds of carbohydrates like
glucose, sucrose and starch. It is not directly involved in the respiration process itself.
So, Statement 2 is not correct.
Endothermic reactions are chemical reactions in which the reactants absorb heat energy from the
surroundings to form products. An exothermic reaction is a reaction in which energy is released in the
form of light or heat.
While photosynthesis is an endothermic reaction where light energy is needed to synthesise food,
respiration is an exothermic reaction where C-C bonds of complex compounds are broken through
oxidation within the cell, leading to the release of a considerable amount of energy. Therefore, an
endothermic reaction through which food is synthesised is not part of the respiration process.
So, Statement 3 is not correct.

55. Consider the following statements:


1. Both Hydrogen and Nitrogen are colourless, odourless and tasteless gases at room temperature.
2. Both Hydrogen and Nitrogen are generally inert gases at standard temperature and pressure.
Which of the statements given above is/are correct?
(a) 1 only
(b) 2 only
(c) Both 1 and 2
(d) Neither 1 nor 2

SIA-A-GS I (43-A)
EXPLANATION:
Hydrogen is the first element in the periodic table. Hydrogen (H) is a colourless, odourless, tasteless,
flammable gaseous substance. It is practically insoluble in water. It is the lightest gas.
At room temperature, Molecular hydrogen can react with many elements and compounds at a very low
reaction rate. At elevated temperatures, however, the reaction rates are high. Hence, hydrogen is not an
inert gas.
Nitrogen (N2) is a colourless, odourless, and tasteless gas that makes up 78.09% (by volume) of the air we
breathe. It is non-flammable, and it will not support combustion. At standard temperature and pressure,
Nitrogen is an inert gas. Nitrogen gas is slightly lighter than air and slightly soluble in water. It is not a very
active element. It combines with relatively few other elements at room temperature. Yet, the compounds of
nitrogen are enormously important both in living organisms and in industrial applications. Hence, Nitrogen
is an inert gas, and hydrogen gas is flammable.
So, Statement 1 is correct, and Statement 2 is not correct.

56. With reference to the Indian Space Policy 2023, consider the following statements :
1. The policy promotes Non-Governmental Entities (NGE) to carry out independent space activities.
2. As per this policy, NewSpace India Limited (NSIL) will focus on research and development of new
space technologies and applications.
3. The policy intends to privatise the Indian Space Research Organization within 5 years.
How many of the statements given above are correct?
(a) Only one
(b) Only two
(c) All three
(d) None
EXPLANATION:
The Indian Space Policy 2023, announced on April 20, 2023, is a framework to increase India's space
capabilities and commercial presence in space. The policy aims to strengthen India's private space sector
by allowing non-government entities (NGEs) to participate in all space activities.
Non-governmental entities shall be allowed to undertake end-to-end activities in the space sector through
the establishment and operation of space objects, ground-based assets and related services, such as
communication, remote sensing, navigation, etc. This would be subject to such guidelines/regulations as
prescribed by IN-SPACe (Indian National Space Promotion & Authorisation Centre). So, Statement 1 is
correct.
The policy made it clear that NewSpace India Limited (NSIL), as the public sector undertaking under the
Department of Space, shall be responsible for commercialising space technologies and platforms created
through public expenditure.
The policy states that ISRO (not NSIL), as the national space agency, will focus primarily on the research
and development of new space technologies and applications and on expanding the human understanding
of outer space. To achieve this goal, the policy said ISRO should carry out applied research and
development of newer systems so as to maintain India's edge in the sector in the areas of space
infrastructure, space transportation, space applications, capacity building and human spaceflight. The
Indian Space Policy (ISP) 2023 does not intend to privatise the Indian Space Research Organization (ISRO)
within five years. Still, it does aim to increase private sector participation in the space sector.
So, Statements 2 and 3 are not correct.

57. With reference to the independence and impartiality of the Election Commission of India, which
of the following statements is/are correct?
1. The Constitution has not prescribed the qualifications of the members of the Election Commission.
2. The Constitution has specified the term of the members of the Election Commission.
3. The Constitution has not debarred the retiring election commissioners from any further
appointment by the government.

SIA-A-GS I (44-A)
Select the correct answer using the code given below:
(a) 1 and 2 only
(b) 2 only
(c) 2 and 3 only
(d) 1 and 3 only
EXPLANATION:
The Election Commission of India is a permanent Constitutional Body. The Constitution of India has made
provision under Article 324 for the constitution of an independent Election Commission (EC) for the
conduct of free and fair elections in the country. An issue of independence and impartiality in the
functioning of the Election Commission. First, the Constitution of India is silent about the qualification
and the tenure of the Chief Election Commissioner of India. Every matter in this connection has been left
to the President, who is guided by provisions of law as enacted by the Parliament.
Hence, The Constitution has not prescribed the qualifications (legal, educational, administrative, or
judicial) of the members of the Election Commission. So, Statement 1 is correct.
The Constitution has not specified the term of the members of the Election Commission. Further, the term
of the members of the Election Commission has been given in the Election Commission (conditions of
service of election commissioners and transaction of business) Act, 1991, which contains the provision
for Members of the Election Commission will hold office for six years, or until they attain the age of 65
years, whichever is earlier.
Members of the Commission cannot be re-appointed. If an EC is appointed as a Chief Election
Commissioner of India, the overall period of the term may not be more than six years. They can also resign
at any time by writing to the President. So, Statement 2 is not correct.
The Constitution has sought to safeguard and ensure the independence and impartiality of the election
commission. One of the flaws can be noted: The Constitution has not debarred the retiring election
commissioners from any further appointment by the government. So, Statement 3 is correct.

58. Consider the following statements regarding the Panchayat Extension to Scheduled Areas (PESA) Act:
1. The Gram Sabha has the power to exercise control over money lending to the Scheduled Tribes.
2. The Gram Sabha has the ownership of minor forest produce.
3. The Gram Sabha has the power to prohibit the sale and consumption of any intoxicant.
Which of the statements given above are correct?
(a) 1 and 2 only
(b) 1, 2 and 3
(c) 2 and 3 only
(d) 1 and 3 only
EXPLANATION:
The Panchayat (Extension of the Scheduled Areas) Act, 1996, or PESA, was enacted by the Centre to
ensure self-governance through gram Sabha (village assemblies) for people living in scheduled areas. It
recognizes the right of tribal communities, who are residents of the Scheduled Areas, to govern themselves
through their systems of self-government and also acknowledges their traditional rights over natural
resources.
The Gram Panchayat or as the case may be, the Gram Sabha shall exercise such powers and perform
such functions in such manner and to such extent as may be prescribed in respect of the following
matters, namely
➢ The power to exercise control over money lending to Scheduled Tribes. Thus, this provision ensures
that financial transactions are conducted fairly and transparently, especially concerning the
Scheduled Tribes.
So, Statement 1 is correct.

SIA-A-GS I (45-A)
The PESA Act also gives power to the gram sabhas over the management of resources over Jal, Jangal,
Zameen (water, forest, and land), the three major demands of tribals; management of village markets,
minor forest produce; mines and minerals; markets; and human resources in the Schedule Areas must
be enforced in the PESA Gram Sabha. So, Statement 2 is correct.
The gram sabha would have the power to monitor and prohibit the manufacturing, transport, sale, and
consumption of intoxicants within their village limits. It also has to maintain peace and resolve conflicts
arising in the village while protecting tribal customs and traditions and encouraging customs like ghotul.
So, Statement 3 is correct.

59. With reference to ‘Sealed Cover Jurisprudence’, consider the following statements:
1. The Supreme Court accepts information from government agencies in sealed envelopes that can
only be accessed by judges.
2. The doctrine of Sealed Cover Jurisprudence is derived from the Indian Penal Code of 1860.
Which one of the statements given above is/are correct?
(a) 1 only
(b) 2 only
(c) Both 1 and 2
(d) Neither 1 nor 2
EXPLANATION:
Sealed Cover Jurisprudence is a practice used by the Supreme Court and sometimes lower courts to ask
for or accept information from government agencies in sealed envelopes that can only be accessed by
judges. So, Statement 1 is correct.
There is no law specifying the sealed cover jurisprudence, but the Supreme Court derives its power from
Rule 7 of Order XIII of the Supreme Court Rules and Section 123 of the Indian Evidence Act of 1872 (not
derived from the Indian Penal Code of 1860).
Rule 7 of order XIII of the Supreme Court Rules deals with the fact that the Chief Justice or court directs
any information that can be kept confidential only if its publication is not in the public interest. As an
exception, no one or party is allowed to access the information except the Chief Justice allows the other
parties to access the information.
Under Section 123 of the Indian Evidence Act of 1872, unpublished official documents about state affairs
are given protection and a public servant cannot be forced to disclose such information. Also, information
can be asked in secrecy or confidence when its publication delays an ongoing investigation. Thus, it is So,
Statement 2 is not correct.

60. Consider the following statements:


1. One-third of a Metropolitan Planning Committee's members should be elected by the elected
members of municipalities and panchayat chairpersons.
2. The Chairperson of the Metropolitan Planning Committee shall forward the development plan to
the Governor of the state.
3. The Metropolitan Planning Committee shall, in preparing the draft development plan, have regard
to the priorities set by the Government of the state only.
How many of the statements given above is/are correct?
(a) Only one
(b) Only two
(c) All three
(d) None
EXPLANATION:
According to the Constitution (74th Amendment) Act, 1992, not less than two-thirds (Not one-third) of the
members of a metropolitan planning committee should be elected by the elected members of the
municipalities and chairpersons of the panchayats in the metropolitan area from amongst themselves.

SIA-A-GS I (46-A)
The representation of these members in the committee should be in proportion to the ratio between the
population of the municipalities and the panchayats in that metropolitan area. So, Statement 1 is not
correct.
The Committee for Metropolitan Planning was formed as per Article 243-ZE (4) of the Constitution. The
Chairperson of every Metropolitan Planning Committee shall forward the development plan, as
recommended by such Committee, to the Government of the State (not the governor of a state). The
Metropolitan Planning Committee shall also consult such institutions and organizations as the Governor
may specify while preparing the draft development plan. So, Statement 2 is not correct.
As per Article 243ZE(3), Every Metropolitan Planning Committee shall, in preparing the draft development
plan, have regard to
➢ The plans prepared by the Municipalities and the Panchayats in the Metropolitan area and matters of
common interest between the Municipalities and the Panchayats, including coordinated spatial
planning of the area, sharing of water and other physical and natural resources, the integrated
development of infrastructure and environmental conservation.
➢ The overall objectives and priorities are set by both the Government of India and the government of
the state. So, Statement 3 is not correct.

61. Which of the following countries are part of the UN’s Champions Group of the Global Crisis Response
Group ?
1. Sweden
2. Denmark
3. India
4. Bangladesh
5. Indonesia
6. Mauritania
Select the correct answer using the code given below :
(a) 1, 2, 5 and 6 only
(b) 2, 3, 4 and 5 only
(c) 3 and 6 only
(d) 1, 2, 3, 4, 5 and 6
EXPLANATION:
The Champions of the Global Crisis Response Group was set up by the UN Secretary-General (UNSG) in
March 2022 to address urgent and critical global issues pertaining to interlinked crises in food security,
energy, and finance and to coordinate a global response. The Champions Group oversees the Global Crisis
Response Group and comprises heads of state and heads of governments of Bangladesh, Barbados,
Denmark, India, Germany, Indonesia, and Senegal.
Recently, India has joined the Champions Group of the Global Crisis Response Group (GCRG), accepting
an invitation from the United Nations Secretary-General. The decision to join the Group reflects India’s
increasing global leadership and commitment to addressing contemporary global challenges. India’s
participation will further boost the efforts of the United Nations in finding result-oriented solutions to
developmental issues that impact the world, particularly developing countries. Thus, Sweden and
Mauritania are not part of the UN’s Champions Group of the Global Crisis Response Group. So, Option
(b) is correct.

62. Consider the following statements with respect to the Acing Development of Innovative Technologies
with the iDEX (ADITI) scheme:
1. The ADITI Scheme under Ministry of Commerce and Industry was launched to promote innovation
in critical and strategic deep-tech technologies by providing grants.
2. Startups recognised by the Department for Promotion of Industry and Internal Trade are eligible
to avail grants.
3. It envisages creating a ‘Technology Watch Tool’ to bridge the gap between expectations and
requirements of the Defence Innovation Ecosystem.
SIA-A-GS I (47-A)
How many of the above statements is/are correct?
(a) Only one
(b) Only two
(c) All three
(d) None
EXPLANATION:
The Acing Development of Innovative Technologies with iDEX (ADITI) scheme is aimed to promote
innovations in critical and strategic defence technologies. Under the scheme, startups are eligible to
receive a grant-in-aid of up to Rs 25 crore for their research, development, and innovation endeavours in
defence technology.
The ADITI scheme worth Rs 750 crore for the period 2023-24 to 2025-26 falls under the iDEX (Innovations
for Defence Excellence) framework of the Department of Defence Production (DDP), Ministry of Defence
(not under the Ministry of Commerce and Industry). It aims to develop about 30 deep-tech critical and
strategic technologies in the proposed timeframe. So, Statement 1 is not correct.
The eligibility to avail of the grants under the ADITI scheme is as follows:
➢ Startups, as defined and recognised by the Department for Promotion of Industry and Internal Trade
(DPIIT), Ministry of Commerce and Industry, Government of India.
➢ Any Indian company incorporated under the Companies Act 1956/2013, primarily a Micro, Small and
Medium Enterprises (MSME) as defined in the MSME Act, 2006.
➢ Individual innovators are also encouraged to apply (research & academic institutions can use this
category to apply). However, post-selection, individual innovators need to register as Startup/MSMEs.
So, Statement 2 is correct.
The ADITI scheme also envisages the creation of a ‘Technology Watch Tool’ to bridge the gap between the
expectations and requirements of the modern Armed Forces and the capabilities of the defence innovation
ecosystem. The “Technology Watch” not only enables the development of domestic capabilities but also
aids in consolidating a strategic policy with regular monitoring of the milestones achieved. So, Statement
3 is correct.

63. With reference to Fishing cats, consider the following statements:


1. It is a nocturnal animal.
2. It is usually found in mangroves and marshes.
3. It is found in the areas of western ghats.
How many of the statements given above is/are correct?
(a) Only one
(b) Only two
(c) All three
(d) None
EXPLANATION:
The fishing cat (Prionailurus viverrinus) is a feline with a powerful build and stocky legs. The size of an
adult ranges from 57-78 cm and weighs between 5-16 kg. The fishing cat is an adept swimmer and enters
water frequently to prey on fish, as its name suggests. It is known even to dive to catch fish. They are
mostly nocturnal, about twice the size of a domestic cat, boast a glossy yellowish-grey double-layered coat,
which offers a water barrier and insulation when it plunges to catch fish; their paws are slightly webbed,
while their ears close shut when their nose is submerged in shallow water. So, Statement 1 is correct.
The fishing cat is specifically adapted to living and hunting in semiaquatic habitats: in marshes, along
rivers and oxbow lakes, in reeds and mangroves. The young start playing in water within two months of
birth. In India, fishing cats are mainly found in the mangrove forests of the Sundarbans, on the foothills
of the Himalayas along the Ganga and Brahmaputra river valleys and in the Western Ghats. So,
Statements 2 and 3 are correct.

SIA-A-GS I (48-A)
64. Asola Bhatti Wildlife Sanctuary was recently in the news for the presence of leopards after many
decades is situated in :
(a) Punjab
(b) New Delhi
(c) Madhya Pradesh
(d) Uttar Pradesh
EXPLANATION:
The Asola-Bhatti Wildlife Sanctuary (ABWS) located in the South district of Delhi has a unique place in
India’s network of protected areas. Asola Bhatti was declared a sanctuary in 1991. It is the only protected
area representing the northeastern, rolling outliers as a landform of the country’s oldest hill ranges, the
Aravallis. As a ‘wildlife’ sanctuary it is very different from the popular perception of a sanctuary. The
sanctuary covers an area of 32 sq km.
The Indian subcontinent is a diverse region with five big cat species: lion, tiger, clouded leopard, snow
leopard, and Indian leopard. The Indian leopard is the most widely distributed and one of nine subspecies
identified through genetic testing. However, the presence and abundance of the leopard in Asola-Bhatti
have been contested over the years. The leopard is one of the nine subspecies found only in the Indian
subcontinent. The presence and abundance of a large carnivore like the leopard (Panthera pardus) in
Asola-Bhatti has been heavily contested over the years.
ABWS is also home to other associated mammals such as nilgai (Boselaphus tragocamelus), golden jackal
(Canis aureus), Indian crested porcupine (Hystrix indica), black-naped hare (Lepus nigricollis), spotted
deer (Axis axis), black buck (Antilope cervicapra), Asian palm civet (Paradoxurus hermaphroditus) and
Indian grey mongoose (Urva edwardsii). So, Option (b) is correct.

65. Consider the following statements:


1. The “Agreement on Investment Facilitation for Development” (IFD) is an initiative of the World
Trade Organization.
2. IFD aims at inducing investment flows by creating legally binding provisions.
3. Recently, India has become a party to IFD negotiations with the aim of attracting more FDI in the
future.
Which of the above statements is/are correct ?
(a) 1 only
(b) 1 and 2 only
(c) 2 only
(d) 1 and 3 only

SIA-A-GS I (49-A)
EXPLANATION:
The Investment Facilitation for Development (IFD) Initiative of the World Trade Organization aims to
develop a global agreement on IFD to improve the investment and business climate and make it easier for
investors in all sectors of the economy to invest, conduct their day-to-day business, and expand their
operations. It strengthens investment climates to grow the flow of foreign direct investment (FDI) to foster
economic growth and sustainable development. The IFD agreement, finalised in November 2023, has been
supported by over 70% of WTO member countries, aiming to establish legally binding provisions for
facilitating investment flows. The provisions are:
➢ Transparency of investment measures;
➢ Streamlining and speeding up of administrative procedures;
➢ Providing focal points, domestic regulatory coherence, and cross-border cooperation;
➢ Sustainable investment. So, Statements 1 and 2 are correct.
India’s foremost assertion is that there is no mandate to conduct negotiations on investment. India and
South Africa played a crucial role in not letting the IFD agreement become a part of the WTO rulebook.
India does not seem to be exceedingly concerned about the text of the IFD agreement.
The IFD Agreement, among other things, will require states to augment regulatory transparency and
streamline administrative procedures to bolster foreign investment inflows. Importantly, this agreement
does not contain provisions on market access, investment protection, and investor-state dispute
settlement (ISDS). ISDS, which allows foreign investors to bring treaty claims against the state admitting
investment, has been a contentious issue in recent years. Given the existing structure of the WTO’s
dispute settlement mechanism, where only states can bring legal claims against other states, it is
implausible that ISDS can be a part of it. Therefore, India is not a party to IFD negotiations.
So, Statement 3 is not correct.

66. Greening India's Wastelands with Agroforestry (GROW) Report and portal was launched by :
(a) Ministry of Agriculture and Farmers Welfare
(b) Ministry of Environment, Forest and Climate Change
(c) NITI Aayog
(d) Indian Environmental Society
EXPLANATION:
➢ India was the first country in the world to form and announce the National Agroforestry Policy in 2014,
which focuses on enhancing productivity, profitability, diversity, and ecosystem sustainability.
➢ Agroforestry is a collective name for land-use systems involving trees combined with crops and/or
animals on the same unit of land
➢ Greening and Restoration of Wasteland with Agroforestry (GROW) report and portal is launched by
NITI Aayog. It is a multi-institutional effort led by NITI Aayog to utilise remote sensing and GIS to
assess agroforestry suitability across all districts in India. Using thematic datasets, an Agroforestry
Suitability Index (ASI) was developed for national-level prioritisation. The report provides state-wise
and district-wise analysis, supporting government departments and industries for greening and
restoration projects.
➢ The "Greening and Restoration of Wasteland with Agroforestry (GROW)-Suitability Mapping" portal-
Bhuvan allows universal access to state and district-level data.
➢ The GROW initiative aligns with national commitments, aiming to restore 26 million hectares of
degraded land by 2030 and create an additional carbon sink of 2.5 to 3 billion tonnes of carbon dioxide
equivalent.
So, Option (c) is correct.

SIA-A-GS I (50-A)
67. Consider the following statements with regard to the "DigiReady Certification “( DRC) portal, recently
seen in the news :
1. It is launched by the Quality Council of India and the Open Network for Digital Commerce.
2. It aims at fostering digital inclusion in the MSME sector.
Which of the above statements is/are correct ?
(a) 1 only
(b) 2 only
(c) Both 1 and 2
(d) Neither 1 nor 2
EXPLANATION:
➢ The Quality Council of India (QCI) and Open Network for Digital Commerce (ONDC) launched the
DigiReady Certification (DRC) portal under the Ministry of Commerce & Industry to foster digital
inclusion in the MSME sector.
➢ This initiative aims to assess and certify the digital readiness of MSME entities.
➢ By leveraging this online self-assessment tool, MSMEs can evaluate their preparedness to seamlessly
onboard as sellers on the ONDC platform, thereby expanding their digital capabilities and business
potential.
➢ The DRC portal is meticulously designed to facilitate a streamlined seller journey, ensuring that
MSMEs and small retailers can integrate seamlessly into existing digitized workflows. The certification
process evaluates various aspects of digital readiness, including the presence of necessary
documentation for online operations, proficiency in using software and technology, integration with
existing digitised workflows, and efficient management of orders and catalogue offerings.
➢ Once entities become DigiReady and grant their consent, Seller Applications will reach out to them to
facilitate onboarding onto their respective applications. This initiative presents additional business
prospects for sellers, allowing them to become integral to the digital ecosystem. So, Both Statements
1 and 2 are correct.

68. Consider the following statements regarding the Indian toy industry :
1. The toy industry witnessed a 50 % decline in imports and more than a 200% increase in exports
in the last decade.
2. Under the Toys (Quality Control) Order 2020, toys had been brought under Compulsory Bureau
of Indian Standards (BIS) certification with effect from 2021.
3. In India, It is mandatory for children’s toys to conform to 7 Indian Standards and bear the ISI
Mark for the safety of toys.
How many of the above statements is/ are correct ?
(a) Only one
(b) Only two
(c) All three
(d) None
EXPLANATION:
The Government provides all-round support for creating a conducive manufacturing ecosystem for the toy
industry. Some of the measures include promoting Made in India toys; designing toys based on Indian
values, culture, and history; using toys as a learning resource; organising hackathons and grand
challenges for toy designing and manufacturing; monitoring the quality of toys; restricting imports of sub-
standard and unsafe toys and promoting indigenous toy clusters.
As a result of the various measures taken by the Government, the manufacturing ecosystem in Indian Toy
industry has witnessed remarkable growth, leading to substantial decrease of 52% in overall import of
toys from USD 332.55 million in Financial Year 2014-15 to USD 158.7 million in Financial Year 2022-23
and increase of 239% in the exports of toys from USD 96.17 million in Financial Year 2014-15 to USD
325.72 million in Financial Year 2022-23.
So, Statement 1 is correct.
SIA-A-GS I (51-A)
Steps taken by the Government to further enhance the exports of Made in India toys are
➢ The Government issued a Toys (Quality Control) Order, 2020, through which toys had been brought
under compulsory Bureau of Indian Standards (BIS) certification with effect from 01.01.2021. This
QCO applies to both domestic manufacturers as well as foreign manufacturers who intend to export
their toys to India.
So, Statement 2 is correct.
➢ Bureau of Indian Standards, the national standards body of India, has published 10 Indian Standards
on safety aspects of Toys related to physical safety, safety against chemicals, flammability, electrical
safety, etc. These standards prevent the use of unsafe and toxic materials in the manufacturing of
toys.
➢ Out of these 10 standards, 7 are part of the Quality Control Order (QCO) on ‘Safety of Toys’. This
Quality Control Order makes it mandatory that toys for children under 14 years of age conform to 7
Indian Standards (List attached) for the Safety of Toys and bear a Standard Mark (ISI Mark) under
licence from BIS. This Government guideline came into force on 1st Jan 2021.
➢ BIS grants licences to toy manufacturing units based on an assessment of their manufacturing and
testing capability through factory visits as well as testing of toys in a BIS Lab or BIS Recognized Lab
as per the Indian Standards. No person is permitted to manufacture, import sell or distribute, store,
hire, lease or exhibit for sale toys which do not conform to the Indian Standard and do not bear BIS
Standard Mark, i.e. “ISI Mark” under a licence from BIS.
So, Statement 3 is correct.
➢ Under the recent Free Trade Agreements (FTA), including the India-UAE Comprehensive Partnership
Agreement (CEPA) and India-Australia Economic Cooperation and Trade Agreement Trade (ECTA), the
partner countries are providing zero-duty market access for exports of Indian toys.
➢ Directorate General of Foreign Trade (DGFT) in 2019 mandated sample testing of each consignment
and no permission for sale unless the quality testing is successful. In case of failure, the consignment
is either sent back or destroyed at the cost of the importer.
So, Option(c) is correct.

69. Recently, the UNDP’s National Carbon Registry has been accredited as a Digital Public Good. What
is/are the implications of such an initiative?
1. It restricts countries from replicating the information from the registry.
2. It allows countries to modify the registry’s software to tailor solutions, thereby cutting production
costs.
Select the correct answer using the code given below :
(a) 1 only
(b) 2 only
(c) Both 1 and 2
(d) Neither 1 nor 2
EXPLANATION:
UNDP has developed open-source software that allows countries to effectively manage national data and
processes for trading carbon credits. The software, called the National Carbon Registry, has recently been
accredited as a digital public good (DPG).
As a Digital Public Good, the registry uses open-source code, which allows countries to replicate and adapt
the information to fit their own needs and contexts. Thus, it does not restrict to replicate of information
from the registry.
➢ A digital public good is defined by the UN Secretary-General’s Roadmap for Digital Cooperation as a
digital public good as open-source software, open data, open AI models, open standards, and open
content that adhere to the DPG Standard and are of high relevance for the attainment of the UN’s 2030
Sustainable Development Goals (SDGs).
So, Statement 1 is not correct.
The registry’s modules, software, and technical documentation can be reused and tailored by countries,
which could potentially reduce production costs and implementation timelines. Additionally, it can also
be integrated with national measurement, reporting, and verification (MRV) systems and international
digital systems such as UNDP’s voluntary cooperation platform and the global platform Climate Action
Data Trust (CAD Trust) launched by the World Bank.
Thus, it empowers countries to customize the registry while contributing to a shared goal of addressing
climate challenges. So, Statement 2 is correct.

SIA-A-GS I (52-A)
70. With reference to the Indigenous Prototype Fast Breeder Reactor (PFBR), consider the following
statements:
1. It is a machine designed by the Indira Gandhi Centre for Atomic Research which produces more
nuclear fuel than it consumes.
2. It marks the start of stage III of India’s three-stage nuclear power programme.
3. It uses liquid sodium as a coolant which is a highly reactive substance.
How many of the statements given above are correct?
(a) Only one
(b) Only two
(c) All three
(d) None
EXPLANATION:
The prototype fast breeder reactor (PFBR) is a 500 MWe sodium-cooled fast breeder nuclear reactor being
constructed at Kokkilamedu, near Kalpakkam. The Indira Gandhi Centre for Atomic Research (IGCAR)
designed the reactor based on the decades of experience gained from operating the lower-power Fast
Breeder Test Reactor (FBTR). The PFBR has been developed by BHAVINI (Bharatiya Nabhikiya Vidyut
Nigam Limited), a government enterprise set up in 2003 under the Department of Atomic Energy (DAE) to
focus on fast breeder reactors. Construction began in 2004, and the reactor was originally expected to be
completed in September 2010, but it faced a series of delays. It is now scheduled to be put into service in
December 2024. The PFBR is a machine that produces more nuclear fuel than it consumes. So,
Statement 1 is correct.
Recently, Its core-loading event has been hailed as a "milestone" because the operationalisation of the
PFBR will mark the start of stage II of India's three-stage nuclear power programme. In the first stage
nuclear power programme, India used pressurised heavy water reactors (PHWRs) and natural uranium-
238 (U-238), which contain minuscule amounts of U-235, as the fissile material. So, Statement 2 is not
correct.
The PFBR is designed to produce more Pu-239 than it consumes. It uses liquid sodium, a highly reactive
substance, as a coolant in two circuits. Coolant in the first circuit enters the reactor and leaves with
(heat) energy and radioactivity. Via heat exchangers, it transfers only the heat to the coolant in a
secondary circuit. The latter transfers the heat to generators to produce electricity. So, Statement 3 is
correct.

71. Recently, the terms ‘Amaterasu’ and ‘Oh-My-God’ are seen in the news, in the context of :
(a) High energy cosmic rays from a mysterious source
(b) Literary works shortlisted for the Nobel Prize
(c) Non-Fungible Tokens
(d) Strategies followed in Formula 1 motorsport.
EXPLANATION:
Recently, scientists have detected one of the most powerful cosmic rays ever slamming into Earth, but
they have no idea what caused it or where it came from. The extremely high-energy particle has been
named Amaterasu after the Japanese Sun goddess, and it seemingly arrived from a void in space where
nothing is known to exist.
The Amaterasu particle has an energy exceeding 240 exa-electron volts (EeV). That is millions of times
more powerful than the particles produced by the Large Hadron Collider, which is the most powerful
accelerator ever built. It is second only to the "Oh-My-God" particle, another high-energy cosmic ray
detected in 1991. That came in at 320 EeV. So, Option (a) is correct.
The Nobel Prizes were started by Alfred Nobel, a wealthy Swedish 19th-century businessman, inventor
and chemist. In his will, Nobel directed that his home must be utilised to fund "prizes to those who, during
the preceding year, have conferred the greatest benefit to humankind". The first Nobel Prizes were given

SIA-A-GS I (53-A)
five years after his death in 1901. The Nobel Prize in Literature 2023 was awarded to Jon Fosse for his
"innovative plays and prose which give voice to the unsayable". So, Option (b) is not correct.
An NFT or non-fungible token is a unique digital asset stored on the blockchain that serves as proof of
ownership or authenticity for a digital or physical item/right. Unlike fungible assets, NFTs are one-of-a-
kind and cannot be replaced. They use blockchain technology to provide decentralised, secure, and
transparent records of ownership and transfers." So, Option (c) is not correct.
In Formula 1 racing, the undercut and overcut are strategic manoeuvres used to gain an advantage over
a rival during pit stops. Through strategic pit stops, racing teams can utilise these tactics to gain a lead
over them and enhance their ranking on the racing circuit. So, Option (d) is not correct.

72. Consider the following statements about the XpoSat mission launched by ISRO:
1. It is India’s first dedicated polarimetry mission to study various dynamics of bright astronomical
X-ray sources in extreme conditions.
2. A polarimeter is a scientific instrument that utilises the principles of wave superposition to make
precise measurements by analysing the interaction of waves.
3. The mission has an instrument designed to operate in the medium X-ray energy range of 8 to 30
kiloelectron Volts.
How many of the above statements are correct?
(a) Only one
(b) Only two
(c) All three
(d) None
EXPLANATION:
XPoSat (X-ray Polarimeter Satellite) is India's first dedicated polarimetry mission to study various
dynamics of bright astronomical X-ray sources in extreme conditions. The spacecraft will carry two
scientific payloads in a low earth orbit.
XPoSat comprises two payloads, including Indian X-ray Polarimeter (POLIX) and X-ray Spectroscopy and
Timing (XSPECT). They have been built by Raman Research Institute (RRI) and UR Rao Satellite Centre,
both located in Bengaluru.
So, Statement 1 is correct.
Interferometry (Not polarimeter) refers to techniques that use superimposed waves to extract information
about the waves. It uses the interference these waves experience to make accurate measurements of the
waves. It is used in many areas of science, such as astronomy, engineering, oceanography, physics, and
fibre optics. Popular applications of interferometry in industry include the measurement of small
displacements, refractive index changes, and surface irregularities.
The polarimeter is a kind of instrument for measuring the optical rotation of a substance. It is an
instrument which measures the angle of rotation by passing polarized light through an optically active
(chiral) substance. Through measuring the optical rotation, the polarimeter can be used to analyze the
concentration, content, and purity of a substance and to calculate the value of the solution. XPoSat is
the world's second satellite-based mission dedicated to making X-ray polarimetry measurements.
So, Statement 2 is not correct.
POLIX is the world's first instrument designed to operate in the medium X-ray of 8 to 30 kilo electron
Volt (keV) energy band. It comprises a collimator, which is the key component to filter light originating
from bright sources in the field of view. Moreover, there is a scatterer consisting of four X-ray proportional
counter detectors (that prevent the trapped light from escaping). It will observe a few tens of astronomical
sources. It was conceived, designed, and built at Raman Research Institute (RRI).
So, Statement 3 is correct.

73. With reference to Voice over New Radio (VoNR), Consider the following statements :
1. VoNR is a high-speed wireless communication standard for 5G networks.
2. VoNR has lower latency than VoLTE.
3. VoNR is backward compatible with 4G phones.
SIA-A-GS I (54-A)
How many of the above statements are correct?
(a) Only one
(b) Only two
(c) All three
(d) None
EXPLANATION:
Voice over 5G, also known as Voice over New Radio (VoNR), is probably the future of voice calling. This
standard allows voice calls over 5G networks instead of the current standard that uses 4G. In simple
terms, Vo5G takes all the improvements of 5G – speed, capacity, responsiveness – and applies them
squarely to voice. So, Statement 1 is correct.
VoNR with 5G’s substantially higher bandwidth and lower latency compared to 4G LTE, VoNR calls
benefit in a few key areas. First, call quality is better with VoNR. It utilises more advanced audio codecs
that provide superior clarity and fidelity based on 5G’s increased data capacity. Second, call connection
time should be faster with VoNR. Additionally, the reliability and continuity of calls should be better
under VoNR. There is lower packet loss, potentially leading to fewer voice cutouts. So, Statement 2 is
correct.
To use voice calling over 5G (Vo5G), one needs three things: a phone that supports Vo5G, a carrier that
offers Vo5G, and a 5G signal in your area. VoNR is not backwards compatible with 4G phones. Backward
compatibility refers to the ability of a newer device or software version to work seamlessly with older
versions or devices. So, Statement 3 is not correct.

74. With reference to the FASTER system of the Supreme Court of India, which of the following statements
is correct?
(a) It is a national repository of data related to cases instituted, pending and disposed of by the courts
across India.
(b) It provides a repository for judgments and final orders of the High Court.
(c) It is a digital platform to communicate orders of the Supreme Court to concerned
authorities through a secured electronic communication channel.
(d) It is the first AI portal to leverage machine learning in dealing with huge chunks of case data in
the Supreme Court.
EXPLANATION:
The flagship project called the 'e-Courts project' completed the full circle with the Supreme Court of India
onboarding the National Judicial Data Grid (NJDG). The National Judicial Data Grid portal is a national
repository of data relating to cases instituted, pending, and disposed of by the courts across the length
and breadth of the country. All three tiers of the Indian judiciary (the Supreme Court of India, the High
Courts and subordinate courts at district, municipal and village levels) are now on the National Judicial
Data Grid portal. Thus, the given statement is not related to the FASTER system of the Supreme Court
of India. So, Option (a) is not correct.
In April 2021, A new ‘Judgment & Order Search’ portal, an initiative of the Department of Justice,
Ministry of Law and Justice, was inaugurated by the Supreme Court of India for the convenience of its
stakeholders in searching judgments easily. The new portal for ‘judgments search’ is set to provide a
repository for Judgments and Final Orders of the High Courts. Thus, the given statement is not related
to the FASTER system of the Supreme Court of India. So, Option (b) is not correct.
The Supreme Court of India launched a digital platform to ensure that all its orders on bail and the
release of prisoners are digitally transmitted straight to jails and concerned high courts through a
secured network that will involve no third-party interference. The orders passed by the Supreme Court
or any court can now be transmitted safely without interference by any party.
The software, called 'Fast and Secured Transmission of Electronic Records' (FASTER), will be used to
communicate interim orders, stay orders, and bail orders passed by the top court to the concerned
authorities through a secured electronic communication channel. So, Option (c) is correct.
In April 2021, the Supreme Court of India launched its first AI portal, the Supreme Court Portal for
Assistance in Courts Efficiency (SUPACE), to leverage machine learning in dealing with huge chunks of
SIA-A-GS I (55-A)
case data. It was built by ManCorp Innovation Labs; the tool collects relevant facts and laws and makes
them available to a judge. The use of AI will streamline, reduce costs, and shorten the process of
exercising the right to get justice. Since it has the power to lower delays and case pendency, it will
transparently and economically improve the service delivery process. Thus, the given statement is not
related to the FASTER system of the Supreme Court of India. So, Option (d) is not correct.

75. In the context of the Indian economy, which of the following situations is likely to result in a decrease
in the foreign exchange reserves?
1. Quantitative tightening by the US Federal Reserve Bank
2. Capital flight during market volatility
3. Increase in the value of gold
Select the correct answer using the code given below:
(a) 1 and 2 only
(b) 2 and 3 only
(c) 1 and 3 only
(d) 1, 2 and 3
EXPLANATION:
The US Federal Reserve raises the interest rate (Quantitative tightening), to increase the cost of credit
throughout the economy. In simple terms, higher interest rate makes loans more expensive for all, be it
businesses or consumers, and eventually, everyone ends up spending more on interest payments.
After a rate hike by the US Federal Reserve, the difference between interest rates in the US and India
shrinks which affects the currency trade negatively. This starts a chain reaction; the US rate hike leads
to an increase in interest rates in India. Therefore, foreign investors will be tempted to withdraw from the
Indian market and invest in US assets as the Dollar and the US Treasury yield become more attractive in
the US and the Indian market begins to see Capital flight.
This makes the rupee weaker and it prompts RBI for a rate hike in India. If the rupee falls significantly,
the RBI may be forced to sell some dollars to help shore up the domestic currency. This depletes the
domestic Forex reserve. So, Statements 1 and 2 are correct.
➢ Recently, India's foreign exchange reserves decreased by $462 million to $590.3 billion, despite an
increase in foreign currency assets. The decline was due to a decrease in the value of gold held by the
Reserve Bank of India (RBI).
➢ Gold reserves are a component of foreign exchange reserves, but their valuation changes do not
necessarily lead to a decrease in overall reserves. In fact, rising gold prices can enhance the value of
foreign exchange reserves. So, Statement 3 is not correct.

76. Consider the following :


1. Capacity building and skill development of farmers
2. Increased access to resources like credit, inputs and technologies
3. Better income and livelihoods of farmers
4. To get the support of market intermediaries to market their products
How many of the above are not the likely advantages of promoting Farmer Producer Organisations in
India ?
(a) Only one
(b) Only two
(c) Only three
(d) All four
EXPLANATION:
A Farmer Producer Company (FPC) can be formed by any 10 or more primary producers or by two or more
producer institutions, or by a contribution of both. An FPC is a hybrid between cooperative societies and
private limited companies. The Farmer Producer Companies, registered under the Indian Companies Act

SIA-A-GS I (56-A)
2013, have democratic governance; each producer or member has equal voting rights irrespective of the
number of shares held.
FPOs typically facilitate activities such as collective marketing, input procurement, value addition, and
accessing financial services. By pooling resources and leveraging economies of scale, FPOs enable
smallholder farmers to overcome individual constraints and achieve better prices for their produce.
Additionally, FPOs often play a vital role in capacity building, knowledge dissemination, and fostering
sustainable agricultural practices within their communities. Overall, FPOs serve as catalysts for rural
development and empowerment, contributing to the socio-economic upliftment of farmers and rural
communities. So, Statement 1 is correct.
In India, the necessity for an FPO arose as a result of farmers’ struggles to get their goods into major
markets. FPO helps small farmers improve the quality of their food by giving them quality seeds, teaching
farmers how to manage soil, use less fertiliser, manure, irrigation, and livestock. Along with assisting in
the selling of goods to larger and better markets, it also updates and aids farmers in implementing cutting-
edge farming equipment and upgrades in technology. So, Statement 2 is correct.
The main aim of FPC is to ensure better income for the producers through an organisation of their own.
Small producers do not have the volume individually (both inputs and produce) to get the benefit of
economies of scale. So, Statement 3 is correct.
In agricultural marketing, there is a long chain of intermediaries who very often work non-transparently
leading to the situation where the producer receives only a small part of the value that the ultimate
consumer pays. The FPOs will help to eliminate the chain of intermediaries in agricultural marketing.
Through aggregation, the primary producers can avail the benefit of economies of scale. They will also
have better bargaining power vis-à-vis the bulk buyers of produce and bulk suppliers of inputs. So,
Statement 4 is not correct.

77. With reference to the Taiga forest, consider the following statements:
1. Unlike deciduous trees in temperate forests, coniferous trees in taiga forests do not lose their
leaves in winter.
2. It is located predominantly in the Northern Hemisphere, which experiences a dry climate due to
high evaporation rates and low annual precipitation.
3. Clearcutting, the most popular type of logging in Taigas, increases the risk of soil erosion and
flooding.
4. Taiga soil is acidic due to the breakdown of coniferous tree needles.
Which of the statements given above are correct?
(a) 1 and 3 only
(b) 2 and 4 only
(c) 1, 2 and 4 only
(d) 1, 3 and 4 only
EXPLANATION:
The taiga is a forest of the cold, subarctic region. The subarctic is an area of the Northern
Hemisphere that lies just south of the Arctic Circle. The taiga lies between the tundra to the north
and temperate forests to the south. Taigas are thick forests.
Coniferous trees, such as spruce, pine, and fir, are common. Coniferous trees have needles instead
of broad leaves, and their seeds grow inside protective, woody cones.
While deciduous trees of temperate forests lose their leaves in winter, conifers never lose their needles.
For this reason, conifers are also called "evergreens."(because the majority of conifer plants retain their
foliage all year round by shedding and growing new ones simultaneously).
Conifers have adapted to survive the long, cold winters and short summers of the taiga.
Their needles contain very little sap, which helps prevent freezing. Their dark colour and triangle-shaped
sides help them catch and absorb as much of the sun's light as possible. Therefore, Unlike deciduous
trees in temperate forests, coniferous trees in taiga forests do not lose their leaves in winter. So,
Statement 1 is correct.
SIA-A-GS I (57-A)
The taiga forest is predominantly found in the Northern hemisphere, roughly between 50° and 60° north
latitude that covers most of inland Alaska, Canada, Sweden, Finland, inland Norway, and Russia
(especially Siberia), as well as parts of the extreme northern continental United States, northern
Kazakhstan, and Japan (Hokkaidō). They are known as boreal forests in North America. It is the world's
largest terrestrial biome, occupying roughly an estimated one-fifth to one-third of the world's forest land.
Taiga biome has cold, dry winters and short, cool, wet summers. The annual precipitation is from 40 cm
to 100 cm and usually takes the form of snow. As evaporation is also low for most of the year (Not high
evaporation), precipitation exceeds evaporation and is sufficient for dense vegetation growth. Snow may
remain on the ground for as long as nine months in the northernmost extensions of the taiga ecozone. So,
Statement 2 is not correct.

Clearcutting is the most popular type of logging in taigas. Clearcutting involves cutting down all
the trees in a designated area. This destroys habitats for many organisms that live in and around the trees
and makes it difficult for new trees to grow. Clearcutting also increases the risk of erosion and flooding in
the taiga. Without a root system to anchor it, a taiga's soil can be blown away by wind or worn away by
rain or snow. This exposes the bedrock and permafrost beneath the taiga, which does not support many
forms of life. So, Statement 3 is correct.
The soil in the Taiga Forest has low fertility as water leaches through the upper layer of sandy soil quickly,
dragging almost all available nutrients with it.. Most of the nutrients are only in a thin layer on the surface
of the ground. The soil is also acidic. This is mainly due to the breakdown of needles that fall from
coniferous trees. So, Statement 4 is correct.

78. With reference to Tiger Reserves, consider the following statements:


1. As per the All India Tiger Estimation – 2022, Corbett National Park has the highest abundance of
tigers.
2. The Simlipal Tiger Reserve is the only home for melanistic tigers in the wild in the world.
3. Among the 55 Tiger Reserves in India, Nagarjunsagar Srisailam Tiger Reserve has the largest area
of core/critical tiger habitat.
How many of the statements given above is/are correct?
(a) Only one
(b) Only two
(c) All three
(d) None
EXPLANATION:
India's tiger population increased to 3,682 in 2022, up from 2,967 in 2018, according to an estimate. In
2022, the maximum number of tigers, 785, were reported to be in Madhya Pradesh, followed by

SIA-A-GS I (58-A)
Karnataka (563), Uttarakhand (560), and Maharashtra (444). Nearly a quarter of the tigers were
reportedly outside protected areas. India's tigers are largely concentrated in 55 dedicated tiger reserves
spread across 75,796 square km, spanning about 2.3% of India's total land area. The reserves with the
maximum number of tigers were at the Corbett National Park in Uttarakhand, which reported 260
animals, followed by Bandipur (150) and Nagarhole (141), both in Karnataka.
So, Statement 1 is correct.
Melanism is a genetic condition in which an increased production of melanin, a substance in the skin that
produces hair, eye, and skin pigmentation, results in black (or nearly black) skin, feathers, or hair in an
animal. Many royal Bengal tigers of Similipal belong to a unique lineage with higher-than-normal levels of
melanin, which gives them black and yellow interspersed stripes on their coats. These tigers are not
entirely black and are, therefore, more accurately described as being pseudo-melanistic. Simlipal Tiger
Reserve is the only habitat where these cats are found in the wild in the world. More than 60% of Simlipal's
tigers are 'black', according to Govt data.
So, Statement 2 is correct.
The Government of India pioneered the conservation of our national animal, the tiger, by introducing
"Project Tiger" in 1973. Since its inception, Project Tiger has expanded its coverage to 55 tiger reserves
across the country. Nagarajunasagar Srisailam Tiger Reserve (NSTR) is located in the Nallamala hill ranges
(an offshoot of the Eastern Ghats) of Andhra Pradesh. It attained the status of a Tiger Reserve in 1983. The
total area of the reserve, including the core/critical tiger reserve and buffer, is 3727.82 Square Kilometres,
and it is India's largest Tiger Reserve. Therefore, among 55 Tiger Reserve, The area of Core/Critical Tiger
Habitat of Nagarajunasagar Srisailam Tiger Reserve is 2595.72 Sq.km, which is the largest.
So, Statement 3 is correct.

79. Arrange the following landforms in India from North to South:


1. Bastar Plateau
2. Dafla Hills
3. Parasnath Hills
4. Cardamom Hills
Select the correct answer using the code given below:
(a) 1-2-3-4
(b) 2-3-1-4
(c) 4-1-2-3
(d) 3-2-1-4
EXPLANATION:
➢ Dafla Hills is a steep region on the boundary of western Arunachal and Assam that is home to an
autonomous tribe known as the Daphla. It is located to the north of the subdivisions of Tezpur and
North Lakhimpur and is bounded on the west by the Aka Hills and on the east by the Abor Range.
➢ Parasnath Hills is a mountain range located in the Giridih district of Jharkhand, India. It is also known
as Shikharji and is considered to be the most sacred of all Jain pilgrimage sites. The Parasnath Hills
are part of the Hazaribagh Plateau and form the highest peak in the state of Jharkhand, with an
elevation of 4,479 ft (1,368 m).
➢ Bastar District is located in the southern part of Chhattisgarh and is situated at a height of 2000 ft
plateau from sea level. The majority of this region is covered with dense forests. It primarily covers the
southern part of Chhattisgarh, including four districts: Kanker, Jagdalpur, Dantewada, and the
western and southwestern parts of Odisha, namely Koraput, Malkangiri, Kalahandi, and Nabarangpur
districts, spread over an area of 89,078 square kilometres.
➢ The Cardamom hills denote the entire Western Ghats that lie south of the Palakkad gap. Straddling
both Kerala and Tamil Nadu, the crest of the hills forms the boundary between the two states. Thus,
the north to south arrangement of given landforms in India is Dafla hills - Parasnath Hills - Bastar
Plateau - Cardamom Hills. So, Option (b) is correct.

SIA-A-GS I (59-A)
80. Consider the following statements with reference to the solar power projects in India:
1. The Jawaharlal Nehru National Solar Mission (JNNSM) aims to achieve 100 GW of solar power
capacity by 2022.
2. The Solar Energy Corporation of India is responsible for implementing solar power projects under
the JNNSM.
3. The Government of India have sanctioned 50 Solar Parks in 12 States across the country under
the Solar Parks Scheme.
4. The Kurnool Ultra Mega Solar Park in Andhra Pradesh is currently the largest operational solar
park in India.
How many of the above statements are correct?
(a) Only one
(b) Only two
(c) Only three
(d) All four
EXPLANATION:
The Jawaharlal Nehru National Solar Mission, also known as National Solar Mission, is one of the eight key
National missions which comprise India’s National Action Plan on Climate Change (NAPCC). The mission was
approved on January 11, 2010, by the government.
The Mission has set the ambitious target of deploying 20,000 MW of grid-connected solar power by 2022,
which was revised to 100GW by 2022 in June 2015.
The Mission will adopt a 3 - phase approach: Phase 1 (up to 2012 - 13), Phase 2 (2013 - 17) and Phase 3
(2017 - 22). The immediate aim of the Mission is to focus on setting up an enabling environment for solar
technology penetration in the country both at a centralised and decentralised level. So, Statement 1 is
correct.
In the implementation of JNNSM Phase II, the Government of India introduced the concept of a viability gap
funding (VGF) mechanism wherein solar projects are developed by developers selected through a transparent,
competitive process on a BOO basis to supply solar power at a pre-determined tariff with support from the
Government in terms of VGF. The Government Guidelines prescribe a ceiling on maximum VGF, which can
be allowed under the specific scheme. Solar Energy Corporation of India Limited (SECI) has been designated
as the implementing agency for this VGF Schemes under JNNSM.
Solar Energy Corporation of India Ltd" (SECI) is a CPSU under the administrative control of the Ministry
of New and Renewable Energy (MNRE), set up on 20th Sept 2011 to facilitate the implementation of the
National Solar Mission (NSM) and achievement of targets set therein. So, Statement 2 is correct.
The Scheme for “Development of Solar Parks and Ultra Mega Solar Power Projects” was rolled out in December
2014, with an aggregate capacity of 20,000 MW. Further, the capacity of the Solar Park Scheme was enhanced
from 20,000 MW to 40,000 MW in March 2017 by 2025-26.
As of 30-11-2023, the Ministry has approved 50 solar parks with an aggregate capacity of around 37,490 MW
in 12 States across the country. In these approved parks, an aggregate capacity of 10,401 MW of solar
projects has been commissioned, out of which 284 MW has been commissioned in the calendar year of 2023.
So, Statement 3 is correct.
India's Bhadla Solar Park is the largest solar power park in the world. Bhadla Solar Park is located in Bhadla,
a dry and sandy region in Rajasthan, and spans 14,000 acres. There are over 10 million solar panels at the
park, which contribute to an operational capacity of 2245MW. Therefore, Bhadla Solar Park is currently the
largest operational solar park in India. So, Statement 4 is not correct.

81. The term 'Pineapple Express' refers to which of the following?


(a) A gigantic expressway project that interfaces the four metropolitan cities in India.
(b) A narrow region of atmospheric rivers that originates in the tropical Pacific Ocean.
(c) The joint efforts to tackle illegal deforestation and associated crimes.
(d) A special parcel express train for carrying perishable commodities during pandemic times.

SIA-A-GS I (60-A)
EXPLANATION:
The Golden Quadrilateral Superhighway is a gigantic expressway project that interfaces the four
significant metropolitan communities in India through the 4 significant National Highways of India.
The Golden Quadrilateral Superhighway refers to the national network which connects the 4 major
industrial and cultural cities of India, which included Delhi, Mumbai, Chennai, and Kolkata. Connecting
4 dots forms the shape of a quadrilateral, because of which is known as a quadrilateral. The primary
objective of the project was the creation of superhighways and a reduction of distance and time on the
road while travelling between the various cities. It is known as the fifth-longest highway in the world and
also the largest project in India. The National Highways Authority of India managed the Project.
Thus, the term 'Pineapple Express' does not refer to a gigantic expressway project that interfaces the four
metropolitan communities in India. So, Option (a) is not correct.
The Pineapple Express is a narrow region of atmospheric moisture that builds up in the tropical Pacific.
Atmospheric rivers are relatively long, narrow regions in the atmosphere that transport most of the water
vapour outside of the tropics. Atmospheric rivers are part of the Earth's ocean water cycle and are tied
closely to both water supply and flood risks. While atmospheric rivers can vary greatly in size and strength,
the average atmospheric river carries an amount of water vapour roughly equivalent to the average flow
of water at the mouth of the Mississippi River.
A well-known example of a strong atmospheric river is called the "Pineapple Express" because moisture
builds up in the tropical Pacific around Hawaii (a place famed for growing pineapples) and can wallop the
U.S. and Canada's West Coasts with heavy rainfall and snow. So, Option (b) is correct.
Project LEAP's overall objective is to support law enforcement agencies' joint efforts to tackle illegal
deforestation and associated crimes. Launched in 2018, LEAP has assisted member countries to more
effectively detect, investigate and prevent forestry crime. LEAP has also promoted collaboration between
law enforcement agencies across the globe, building a solid foundation for sustainability and efficiency.
This project is jointly implemented by INTERPOL and the United Nations Office on Drugs and Crime
(UNODC). Thus, the term 'Pineapple Express' does not refer to the joint efforts to tackle illegal deforestation
and associated crimes. So, Option (c) is not correct.
The 'Kisan Rail' is a service announced in the Union Budget 2020-21. The first such train will run weekly
between Devlali (Maharashtra) to Danapur (Bihar). This train will help in bringing perishable agricultural
products like vegetables and fruits to the market in a short period during pandemic times.
The train with frozen containers is expected to build a seamless national cold supply chain for
perishables, inclusive of fish, meat and milk. This train is a step towards realising the goal of doubling
farmers' incomes by 2022. While the Indian Railways have earlier run single commodity special trains
such as Banana Specials, this will be the first multi-commodity train.
Thus, the term 'Pineapple Express' does not refer to special parcel express trains for carrying perishable
commodities during pandemic times. So, Option (d) is not correct.

82. With reference to inclusion in the list of Scheduled Tribes, consider the following statements:
1. Article 342 of the Constitution of India provides that the President can include any tribe or tribal
community in the list of Scheduled Tribes.
2. The proposal for the amendment in the list of Scheduled Tribes is considered only after the
approval of the Registrar General of India (RGI) and the National Commission for Scheduled Tribes
(NCST).
Which of the statements given above is/are correct?
(a) 1 only
(b) 2 only
(c) Both 1 and 2
(d) Neither 1 nor 2
EXPLANATION:
Article 342 (2) of the Constitution empowers the Parliament, not the President, to include communities in
the Scheduled Tribes list. Article 342(1) state that only those tribes or tribal communities who have been
SIA-A-GS I (61-A)
declared as such by the president through an initial public notification will be considered as Scheduled
Tribes. Any further amendment to the list is to be done through an Act of Parliament. Thus, Article 342(2)
States Parliament may, by law, include in or exclude from the list of Scheduled Tribes, any tribe or tribal
community. So, Statement 1 is not correct.
According to the modalities for inclusion first framed in 1999, the proposal for inclusion must originate
from the respective State or Union Territory government. Following this, the proposal is sent to the Union
Tribal Affairs Ministry, which sends it to the Office of the Registrar General of India (ORGI). If the ORGI
approves the inclusion, the proposal is forwarded to the National Commission for Scheduled Tribes.
Following this, the proposal is sent back to the Union government, which after inter-ministerial
deliberations, introduces it in the Cabinet for final approval to bring in the appropriate amendment to the
Constitution (Scheduled Tribes) Order, 1950. So, Statement 2 is correct.

83. Consider the following statements:


1. Legal rights are granted and upheld by the state
2. Legal rights are enforced by the courts of law
Which of the above statements is/are correct?
(a) 1 only
(b) 2 only
(c) Both 1 and 2
(d) Neither 1 nor 2
EXPLANATION:
Legal rights refer to a set of rights enshrined in the legal system of a government. They are provided as
privileges to the inhabitants of that particular state. They are established and preserved by the legal
framework of the Government, ensuring that the same parties may also reinterpret or amend them.
They originate from a legal framework and are established through laws, court rulings and several
treaties. They include human rights, property rights, economic rights, contractual rights and family
rights. State legal frameworks, laws, regulations and Judicial systems uphold legal rights. So, Statement
1 is correct.
Legal rights are enforced through legal procedures, court systems, and other mechanisms provided by the
legal framework. The remedies for breaching legal rights can be sought by asking for Compensation,
Specific performance, injunction, liquidated and unliquidated damages, penalties and even imprisonment.
Examples are Property rights, Economic rights, and Cultural rights. Therefore, Legal rights are enforced
by the courts of law. So, Statement 2 is correct.

84. With reference to the Weapons of Mass Destruction and their Delivery Systems (Prohibition of
Unlawful Activities) Amendment Bill of 2022, consider the following statements:
1. The bill seeks to prohibit the financing of any activity in relation to weapons of mass destruction
and their delivery systems.
2. The amendment seeks to fulfill India's international obligations to the United National Security
Council and the Financial Action Task Force.
Which of the statements given above is/are not correct?
(a) 1 only
(b) 2 only
(c) Both 1 and 2
(d) Neither 1 nor 2
EXPLANATION:
The Amendment bill seeks to modify the Weapons of Mass Destruction and their Delivery Systems
(Prohibition of Unlawful Activities) Act, 2005. The Weapons of Mass Destruction and their Delivery Systems
(Prohibition of Unlawful Activities) Amendment Bill, 2022 seeks to prohibit financing of any activity in
relation to weapons of mass destruction and their delivery systems. It empowers the Central Government

SIA-A-GS I (62-A)
to freeze, seize, or attach funds or other financial assets or economic resources for preventing such
financing and to prohibit making available funds, financial assets or economic resources for any prohibited
activity in relation to weapons of mass destruction and their delivery systems. So, Statement 1 is correct.
The proposed bill focuses more on the financial bit of activities supporting the Weapon of Mass Destruction
and fits the missing block in fulfilling India's international obligations by adhering to the recommendations
of the Financial Action Task Force (FATF) and the United Nations Security Council's targeted financial
sanctions against the financing of Weapon of Mass Destruction.
Thus, the Weapons of Mass Destruction and their Delivery Systems (Prohibition of Unlawful Activities)
Amendment Bill 2022 seeks to prohibit the funding of such unlawful activities that involve weapons of
mass destruction. So, Statement 2 is correct.

85. With reference to Parens patriae jurisdiction, consider the following statements:
1. It can be invoked by the state in the case of minor children whose rights were denied by
irresponsible or abusive parents.
2. Invoking this jurisdiction, ‘Mother Nature’ has been declared as a ‘living being’ having the status
of a legal entity.
Which of the statements given above is/are correct?
(a) 1 only
(b) 2 only
(c) Both 1 and 2
(d) Neither 1 nor 2
EXPLANATION:
The Doctrine of Parens Patriae, which translates as "parent of the nation," is a legal idea that gives the
state the inherent capacity and authority to serve as a guardian for individuals who cannot care for
themselves. In India, this philosophy reflects the nation's commitment to preserve its citizens' welfare and
interests.
In the case of minor children or physically/ mentally/ differently abled people whose rights were denied
by irresponsible/abusive/careless parents, the state can invoke parens patriae jurisdiction. With
sovereign power, the state can assume parenthood to provide all the rights meant for the protection of
such challenged individuals. So, Statement 1 is correct.
The Madras High Court has invoked the "parens patriae jurisdiction" to declare "Mother Nature" as a
"Living Being" having a legal entity/legal person/juristic person/juridical person/moral person/artificial
person having the status of a legal person, with all corresponding rights, duties, and liabilities of a living
person, in order to preserve and conserve them. The State and Central governments are directed to protect
‘Mother Nature’ and take appropriate steps in this regard in all possible ways. So, Statement 2 is correct.

86. “Polycrack technology” and “torrefaction” are related to which one of the following situations?
(a) Identification of areas for extraction of shale gas
(b) Manufacturing of bio-degradable plastics
(c) Implementation of waste-to-energy facilities
(d) Development of lithium-ion batteries
EXPLANATION:
“Polycrack technology” and “torrefaction” are related to the implementation of waste-to-energy Facilities.
The Waste to Energy Plant, a patented technology called Polycrack, is first-of-its-kind in Indian Railways
and the fourth in India. It is the world’s first patented heterogeneous catalytic process, which converts
multiple feedstocks into hydrocarbon liquid fuels, gas, carbon and water. Polycrack Plant can be fed with
all types of Plastic, Petroleum sludge, Un-segregated MSW (Municipal Solid Waste) with moisture up to
50%, E-Waste, Automobile fluff, Organic waste, including bamboo, garden waste, etc., and Jatropha fruit
and palm bunch.

SIA-A-GS I (63-A)
As instances of stubble burning in Punjab and Haryana rose, the Union Environment Ministry announced
a ₹50 crore scheme to incentivise industrialists and entrepreneurs to set up paddy straw pelletisation and
torrefaction plants.
Paddy straw made into pellets or torrefied can be mixed along with coal in thermal power plants. This
saves coal as well as reduces carbon emissions that would otherwise have been emitted were the straw
burnt in the fields, as is the regular practice of most farmers in Punjab and Haryana. So, Option (c) is
correct.

87. Consider the following states :


1. Mizoram
2. Nagaland
3. Ladakh
4. Uttarakhand
As per ISRO's Landslide Atlas of India 2023, which one of the following is the correct descending
order in terms of total number of landslide events during the 1998-2022 period?
(a) 1-2-3-4
(b) 1-4-2-3
(c) 4-3-2-1
(d) 4-1-3-2
EXPLANATION:
With a steady rise in the number of extreme weather events, especially heavy rainfall capable of triggering
landslides and floods, the Indian Space Research Organisation (ISRO) recently released the Landslide
Atlas of India, a detailed guide identifying landslide hotspots in the country.
Landslides are broadly classified based on the type of materials involved (rock, debris, soil, loose mud),
type of movement of the material (fall, topple, slide, rotational slide or translational slide), and type of flow
of the material. Another category is landslides that spread laterally. Landslides mapped in the ISRO atlas
are mainly event-based and season-based.
ISRO's National Remote Sensing Centre (NRSC), Hyderabad, has created a database of landslide-prone
regions of India based on events during 1998 – 2022, primarily along the Himalayas and the Western
Ghats. The correct sequence in descending order in terms of the total number of landslide events during
the 1998-2022 period as per ISRO's Landslide Atlas of India 2023 is Mizoram, Uttarakhand, Nagaland
and Ladakh. Thus, the correct order is 1-4-2-3. So, Option (b) is correct.

SIA-A-GS I (64-A)
88. Recently, the SVAMITVA Scheme conferred with the National Award for ‘Application of Emerging
Technologies for Providing Citizen Centric Services’ related to :
(a) Using Machine Learning tools to avail real-time information of all Panchayat Raj Institutions in
a district
(b) Using drone and GIS technology for property survey mapping in rural areas
(c) Using Artificial Intelligence to provide digital literacy to villagers
(d) A dedicated Social Commerce mobile application to sell rural products
EXPLANATION:
SVAMITVA (Survey of Villages Abadi and Mapping with Improvised Technology in Village Areas) Scheme, a
Central Sector scheme of the Ministry of Panchayati Raj has been conferred with the prestigious National
Award for e-Governance 2023 (Gold) for Application of Emerging Technologies for Providing Citizen Centric
Services at the 26th National Conference on e-Governance (NCeG) organized by the Department of
Administrative Reforms and Public Grievances (DARPG), Government of India on August 2023.
SVAMITVA Scheme provides ownership to property owners in village-inhabited area by using latest drone
and GIS technology. The Property Cards facilitate monetization of rural residential assets for bank loan and
other financial benefits and help in the settlement of land-related disputes through the preparation of 5cm
accuracy maps. The Scheme is implemented with the collaborative efforts of Survey of India being the
technology implementation partner, State Revenue and Panchayati Raj Departments, and NIC–GIS.
So, Option (b) is correct.

89. With reference to the history of India, the Levalloisian technique refers to:
(a) A method of creating super imposition on paintings by the Mughals
(b) A method of creating stone tools by Palaeolithic Man
(c) A technique used to read inscriptions
(d) A method to identify the age of artefacts
EXPLANATION:
Levalloisian technique is named after the locality of Levallois, a suburb of Paris, from where a particular
type of prepared cores was obtained, this technique is recognised from the production of a new mode of
making tools from a ‘tortoise-shaped core’ from the undersurface of which a single blow could strike a flake
tool. This is also referred to as the “Prepared Core technique.” The Middle Palaeolithic tools are made on
flakes and flake blades produced by flake core, discoid and the specialised Levallois technique. In some
regions, there is a continuity of Late Acheulian lithic tradition with refinement in bifacial flaking, second
marginal retouch, and inclusion of small-sized hand-axes and cleavers. In many regions, there is a
switchover in the use of raw material from coarse-grained rocks like quartzite of the preceding phase to
fine-grained rocks like chert, jasper, chalcedony, agate etc.
So, Option (b) is correct.

90. Consider the following statements with respect to Holistic Progress Card (HPC) :
1. It is a multidimensional report that reflects the progress of each learner in the cognitive, affective,
socio-emotional, and psychomotor domains.
2. It was developed by the PARAKH and introduced by NCERT.
3. The HPC aligns with the National Curriculum Framework for School Education (NCFSE), aiming
to make evaluation more teacher-centric.
Which of the above statements are correct ?
(a) 1 and 2 only
(b) 1, 2 and 3 only
(c) 2 and 3 only
(d) 1 and 3 only

SIA-A-GS I (65-A)
EXPLANATION:
Holistic Progress Card (HPC) aims to provide a 360-degree, multidimensional report of progress that
reflects in detail the progress as well as the uniqueness of each learner in the cognitive, affective, socio-
emotional, and psychomotor domains as well as in acquiring life and learning skills, and values.
The HPC signals a paradigm shift moving towards active, learner-centric and experiential learning
pedagogies that involve the learner in activities and projects related to their context and their interests,
such as gardening, artwork, puppetry, playing indoor and outdoor games, toy-based pedagogy,
storytelling, etc. So, Statement 1 is correct.
The New Education Policy (NEP), established in 2020, proposed redesigning the assessment system of
school students in India. The policy had called for a "holistic progress card (HPC)" to track the progress
made by a student throughout the year.
The National Council for Educational and Research Training (NCERT) has introduced a revolutionary
assessment system, the Holistic Progress Card (HPC), aiming to provide a comprehensive evaluation of
students' progress. It is developed by Performance Assessment, Review, and Analysis of Knowledge for
Holistic Development (PARAKH), a standard-setting body under the NCERT, the HPC has already been
designed for the foundational stage (Classes 1 and 2), the preparatory stage (Classes 3 to 5), and the
middle stage (Classes 6 to 8). So, Statement 2 is correct.
The HPC aligns with the National Curriculum Framework for School Education (NCFSE), aiming to make
evaluation more learner-centric (Not teacher-centric). It assesses various aspects of learning, including
academic performance, cognitive abilities, socio-emotional skills, and creativity, through descriptive forms
of evaluation.
The HPC promotes self-awareness and self-esteem. Students are involved in evaluating not only their
performance but also that of their classmates. The HPC includes opportunities for the child in consultation
with the teacher to set future goals, thus providing direction to both the teacher and the child in terms of
future action. So, Statement 3 is not correct.

91. With reference to Project Taara, consider the following statements:


1. It uses optical fibre cables to provide high-speed, high-capacity connectivity over long distances.
2. It operates in the light-based optical spectrum that unlocks 30 times more data capacity than
radio frequency.
Which of the statements given above is/are correct?
(a) 1 only
(b) 2 only
(c) Both 1 and 2
(d) Neither 1 nor 2
EXPLANATION:
The project, known as Taara, is part of Alphabet's innovation lab called X, also nicknamed the "Moonshot
Factory." Taara is working to bring fast, affordable connectivity to the world using a new approach to
wireless optical communication (WOC) technology.
Taara uses beams of light to deliver high-speed, high-capacity connectivity over long distances. In the
same way that traditional fibre uses light to carry data through cables in the ground, Taara uses light to
transmit the information as very narrow, invisible beams. This beam is sent between two small Taara
terminals to create a link. Taara brings fibre-like speeds to areas where it's not economically viable or too
difficult to install fibre. So, Statement 1 is not correct.
Taara's wireless technology can transmit information at high speeds of up to 20 gigabits per second and
distances up to 20 km. Each Taara link consists of two terminals containing mirrors and sensors that
point, acquire, and track beams of light, ensuring they remain on target. The signal between Taara's
terminals mustn't be interrupted, so they're placed up high on towers, poles, or rooftops. Even if a tower
sways due to wind or a bird flies through the beam's path, technology ensures uninterrupted
communication. As Taara operates in the light-based optical spectrum, it unlocks 30 times more data
capacity than radio frequency. So, Statement 2 is correct.

SIA-A-GS I (66-A)
92. With reference to the Human papillomavirus (HPV), consider the following statements:
1. It is a common sexually transmitted virus that affects the skin, genital areas and throat.
2. A majority of the genital HPV strains are high-risk and can lead to cancers like cervical cancer.
3. ‘Cervavac’ is an indigenously developed vaccine to prevent HPV.
How many of the statements given above are correct?
(a) Only one
(b) Only two
(c) All three
(d) None
EXPLANATION:
Human papillomavirus (HPV) refers to a group of viruses which is a common sexually transmitted
infection. Almost all sexually active people will be infected at some point in their lives, usually without
symptoms. Human papillomavirus can affect the skin, genital area, and throat. So, Statement 1 is
correct.
More than 30 strains of the human papillomavirus (HPV) can affect your genitals. These include harmless
forms of HPV, like those that cause genital warts. Only some types of HPV are considered “high risk”
because they can progress to cervical cancer (HPV 16 and HPV 18, are responsible for most HPV-related
cancers). Getting vaccinated against HPV and receiving regular Pap smears can prevent cervical cancer.
So, Statement 2 is not correct.
Union Minister of Science and Technology announced the scientific completion of Cervavac, India’s first
indigenously developed quadrivalent human papillomavirus (qHPV) vaccine for the prevention of HPV
infection and cervical or HPV cancer. It has undergone rigorous research, clinical trials, and regulatory
approvals to ensure its safety and efficacy. The HPV vaccine is formulated to target the high-risk types of
HPV, including HPV 16 and HPV 18, responsible for causing cervical cancer. So, Statement 3 is correct.

93. Consider the following pairs:


S.No (Components of (Owned by)
India Stack)
1. e-KYC - Ministry of Communications
2. DigiLocker - Ministry of Electronics and
Information Technology
3 UPI - National Payment Corporation of
India
4. Account Aggregator - Reserve Bank of India
How many of the pairs given are correctly matched ?
(a) Only one
(b) Only two
(c) Only three
(d) All four
EXPLANATION:
India Stack is the name used to describe a collection of disparate technology products and frameworks.
The components of this collection are owned and maintained by different agencies.
Aadhaar products such as e-auth and e-KYC are owned by the Unique Identification Authority of India
(UIDAI). e-KYC, often called paperless KYC, is the process of electronically verifying the customer's
credentials. This is mandatory for everyone to avail of the services from any financial institution. It is also
called Aadhaar-based eKYC, as your identity is verified electronically. The service provider can access the
details of your Aadhaar from the Unique Identification Authority of India (UIDAI) database. So, Pair (1) is
not correct.
The Ministry of Electronics and Information Technology owns the Digilocker stack. DigiLocker is a flagship
initiative of the Ministry of Electronics & IT (MeitY) under the Digital India programme. DigiLocker aims
at the 'Digital Empowerment' of citizens by providing access to authentic digital documents to citizens'
SIA-A-GS I (67-A)
digital document wallets. DigiLocker is a secure cloud-based platform for the storage, sharing and
verification of documents & certificates. So, Pair (2) is correct.
The National Payments Corporation of India owns the UPI stack. Unified Payments Interface (UPI) is a
system that powers multiple bank accounts into a single mobile application (of any participating bank),
merging several banking features, seamless fund routing & merchant payments into one hood. It also
caters to the "Peer to Peer" collect request which can be scheduled and paid as per requirement and
convenience. So, Pair (3) is correct.
The Account Aggregator framework is regulated by RBI (with an NBFC-AA license) and its technology
standards and is owned by Reserve Bank Information Technology Private Limited (ReBIT). An Account
Aggregator provides data to a Financial Information User (FIU) from a Financial Information Provider (FIP)
based on the user's explicit Electronic/Digital consent. It helps an individual securely and digitally access
and share information from one financial institution they have an account with to any other regulated
financial institution in the AA network. Data cannot be shared without the consent of the individual. So,
Pair (4) is correct.

94. With reference to Tricholime, consider the following statements:


1. It integrates Trichoderma and Lime into a single product where Trichoderma is a fungal biocontrol
agent while lime neutralises soil acidity.
2. Simultaneous application of Lime and beneficial microorganisms like Trichoderma is not generally
recommended in soil application.
3. It is developed by the Indian Institute of Forest Management.
How many of the above statements are correct?
(a) Only one
(b) Only two
(c) All three
(d) None
EXPLANATION:
ICAR-Indian Institute of Spices Research (ICAR-IISR), Kozhikode (Not by Indian Institute of Forest
Management) have successfully developed a new granular lime-based Trichoderma formulation,
‘Tricholime’, integrating Trichoderma- a fungal biocontrol agent used for controlling a variety of soil-borne
pathogens- and Lime into a single product, making the application easier for the farmers. This lime-based
formulation neutralises the soil acidity while promoting plant growth and shields crops from soil-borne
pathogens, all in a single application. So, Statement 1 is correct and 3 is not correct.
Lime has traditionally been employed to counteract soil acidity, but simultaneous application of Lime and
beneficial microorganisms like Trichoderma is not generally recommended. Farmers have to wait for a
period of two to three weeks before incorporating other beneficial microorganisms in the soil. So,
Statement 2 is correct.

95. With reference to Red pandas, consider the following statements:


1. Red pandas are primarily carnivores and inhabit the region of the Eastern Himalayas.
2. They prefer a solitary and daily lifestyle.
3. They are mainly arboreal species that often live close to water sources.
How many of the statements given above is/are correct?
(a) Only one
(b) Only two
(c) All three
(d) None
EXPLANATION:
The red panda is slightly larger than a domestic cat, with a bear-like body and thick russet fur. The belly
and limbs are black, and there are white markings on the side of the head and above its small eyes. Red
pandas are very skilful and acrobatic animals that predominantly stay in trees. They use their long, bushy

SIA-A-GS I (68-A)
tails for balance and to cover themselves in winter, presumably for warmth. Primarily a herbivore (not
carnivore), the name panda is said to come from the Nepali word ‘ponya,’ which means bamboo or plant-
eating animal.
Almost 50% of the red panda’s habitat is in the Eastern Himalayas. They inhibit in high-altitude forests
of Nepal, India, Bhutan, Myanmar and China. The loss of nesting trees and bamboo is causing a decline
in red panda populations across much of their range because their forest home is being cleared. So,
Statement 1 is not correct.

Red pandas are mostly nocturnal (not diurnal), but they are very active at dusk and dawn. Red pandas
are shy and solitary except when breeding. Females create a nest in trees or bamboo thickets in which
their litter is born. The young remain in the nest for about 3 months and stay with their mother until the
breeding season starts. Males do not take part in rearing the young. So, Statement 2 is not correct.
Red pandas are mainly arboreal species, often living close to water sources and in densely bamboo-covered
areas. In the wild, there are two formally recognised subspecies of the red panda. However, recent studies
suggest they might be two separate species. Ailurus fulgens fulgens, which is slightly smaller in size, is
mostly found in Nepal, along with India, Myanmar, and Bhutan. While Ailurus fulgens refulgens is
primarily found in China. So, Statement 3 is correct.

SIA-A-GS I (69-A)
96. Consider the following statements with respect to the Commission for Air Quality Management:
1. It can undertake and promote research in the field of air pollution.
2. The commission shall furnish an annual report to the Central Pollution Control Board.
3. It can initiate actions based on a complaint made by an individual against an individual.
4. The commission shall collect environmental compensation from farmers
5. Only the National Green Tribunal has the jurisdiction to entertain any dispute arising out of
actions taken by the commission.
How many of the statements given above are correct ?
(a) Only two
(b) Only three
(c) Only four
(d) All five
EXPLANATION:
The Commission for Air Quality Management in National Capital Region and Adjoining Areas has been set
up for Air Quality Management in National Capital Region and Adjoining Areas for better co-ordination,
research, identification and resolution of problems surrounding the air quality index and for matters
connected in addition to that or incidental to it.
The Commission shall perform functions such as
➢ Monitoring the measures taken by the States to prevent stubble burning,
➢ Undertaking and promoting research in the field of air pollution,
➢ Spreading awareness regarding air pollution among various sections of society and promoting
awareness of the collective steps that the public may take through publications, the media, seminars,
and other available means encourage the efforts of non-governmental organisations and institutions
working in the field of air pollution. So, Statement 1 is correct.
The Commission shall furnish to the Central Government (Not the Central Pollution Control Board) an
annual report containing such details of the steps taken, proposals made, research awaited, and other
measures undertaken by it in pursuance of its functions under section 12 in such form and manner as
may be specified by regulations. After that the Central Government shall cause the annual report
furnished under sub-section (1) to be laid before each House of the Parliament. So, Statement 2 is not
correct.
The Commission shall take up matters suo motu, or on the basis of complaints made by any individual,
representative body, or organization functioning in the field of environment, against any individual,
association, company, public undertaking, or local body carrying on any industry, operation or process.
So, Statement 3 is correct.
Powers of the Commission include restricting activities influencing air quality, investigating and
conducting research related to environmental pollution impacting air quality, preparing codes and
guidelines to prevent and control air pollution, and issuing directions on matters including inspections or
regulations, which will be binding on the concerned person or authority. Further, the Commission may
impose and collect environmental compensation from farmers causing pollution by stubble burning. The
central government will prescribe this compensation. So, Statement 4 is correct.
Violation of provisions or orders and directions of the Commission for Air Quality Management will be
punishable with imprisonment of up to five years, or a fine of up to one crore rupees, or both.
No civil court shall have jurisdiction to entertain any suit, proceeding, or dispute pertaining to or arising
out of the actions taken or directions issued by the Commission, except the National Green Tribunal will
hear all appeals against the Commission’s orders. So, Statement 5 is correct.

97. In India, the Food Safety Standards Authority of India notified the use of emamectin, benzoate,
fenpyroximate, hexaconazole, propiconazole, and quinalphos. These chemicals are used as :
(a) fruit-ripening agents
(b) herbicides in agriculture
(c) preservatives in processed foods
(d) pesticides in tea plantation
SIA-A-GS I (70-A)
EXPLANATION:
The Food Safety and Standards Authority of India (FSSAI) has implemented the Maximum Residue Limit
(MRL) for five pesticides commonly used for tea plantations. These pesticides include Emamectin
Benzoate, Fenpyroximate, Hexaconazole, Propiconazole, and Quinalphos. The MRL aims to regulate and
ensure the safe levels of these pesticides in tea production. A maximum residue limit (MRL) is the highest
level of pesticide residue that is legally tolerated in or on food or feed when pesticides are applied correctly
in accordance with Good Agricultural Practice. So, Option (d) is correct.

98. Consider the following salient features of the Royal Indian Naval Mutiny of 1946 :
1. The public support for the rebellion was overwhelming.
2. Both the INA trial and racial discrimination were reasons behind the occurrence of the strike.
3. The Mountbatten Plan was the immediate effect of the mutiny.
Which of the given above statements are correct?
(a) 1 and 2 only
(b) 1 and 3 only
(c) 2 and 3 only
(d) 1, 2 and 3
EXPLANATION:
On February 18, 1946, some 1100 Royal Indian Navy (RIN) ratings of HMIS Talwar went on a strike to
protest against
➢ Racial discrimination (demanding equal pay for Indian and white soldiers)
➢ Unpalatable food
➢ Abuse by superior officers
➢ Arrest of a rating for scrawling ‘Quit India’ on HMIS Talwar
➢ INA trials
➢ Use of Indian troops in Indonesia, demanding their withdrawal.
The rebellious ratings hoisted the tricolour, crescent, and the hammer and sickle flags on the mast of the
rebel fleet. Other ratings soon joined, and they went around Bombay in lorries holding Congress flags,
threatening Europeans and police officers. Crowds brought food to the ratings, and shopkeepers invited
them to take whatever they needed. Therefore, Both the INA trial and racial discrimination were reasons
behind the occurrence of the RIN mutiny of 1946, which had overwhelming public support. So,
Statements 1 and 2 are correct.
The Mountbatten Plan was not the immediate effect of the mutiny. The Mountbatten Plan, which proposed
the partition of British India into two separate dominions, India and Pakistan, was formulated later in
1947 after the mutiny had taken place in 1946. So, Statement 3 is not correct.

99. Consider the statements with reference to parallel Government during the National Movement:
1. It was a significant feature of the Civil Disobedience Movement.
2. The Prati Sarkar in Satara, Maharashtra, became the longest-running and most effective parallel
government.
3. The first established parallel government was in Ballia, Uttar Pradesh, under the leadership of
Chittu Pandey.
How many of the statements given above are correct?
(a) Only One
(b) Only two
(c) All three
(d) None
EXPLANATION:
After Cripps's departure, Gandhi framed a resolution calling for British withdrawal and a non-violent,
non-cooperation movement against any Japanese invasion. The CWC meeting at Wardha (July 14, 1942)

SIA-A-GS I (71-A)
accepted the idea of a struggle. The resolution led to the Quit India Movement. Proposed by Jawaharlal
Nehru and seconded by Sardar Patel, it was to be approved by the All-India Congress Committee meeting
in Gowalia Tank, Bombay, in August 1942.
In the early hours of August 9, 1942, in a single sweep, all the top leaders of Congress were arrested and
taken to unknown destinations. The Congress Working Committee, the All-India Congress Committee,
and the Provincial Congress Committees were declared unlawful associations under the Criminal Law
Amendment Act of 1908. The removal of established leaders left the younger and militant elements to their
initiative and hence, Parallel governments were established in many places.
Therefore, Parallel Government was a significant feature of the Quit India Movement (Not the Civil
Disobedience Movement). So, Statement 1 is not correct.
Parallel governments were established in many places during the Quit India Movement. They are as
follows:
➢ The first established Parallel government was in Ballia, Uttar Pradesh (in August 1942 for a week),
under the leadership of Chittu Pandey. He got many Congress leaders released. He was described as
the "Tiger of Ballia" by Jawaharlal Nehru and Subhas Chandra Bose. So, Statement 3 is correct.
➢ Tamluk (Midnapore, from December 1942 to September 1944)—Jatiya Sarkar undertook cyclone relief
work, sanctioned grants to schools, supplied paddy from the rich to the poor, organised Vidyut Vahinis,
etc.
➢ Satara (mid-1943 to 1945)—named "Prati Sarkar", was organised under leaders like Y.B. Chavan,
Nana Patil, etc. Village libraries and Nyayadan Mandals were organised, prohibition campaigns were
carried on, and 'Gandhi marriages' were organised. It was the longest and most successful parallel
government as it lasted for three years. So, Statement 2 is correct.

100. With reference to the Summary Settlement of 1856, consider the following statements:
Statement-I:
This was the first British revenue settlement introduced in Awadh after its annexation by the East
India Company.
Statement-II:
The Settlement system undermined the position and authority of the taluqdars of Awadh by
dispossessing them of their holdings.
Statement-III:
The system reduced the level of exploitation of peasants in Awadh.
Statement-IV:
During the Revolt of 1857, the taluqdars and their peasants in Awadh fought together against the
British.
Which one of the following is correct in respect of the above statements?
(a) Statement-I is incorrect, but Statement-II is correct.
(b) Statement-II and Statement-III are correct, and Statement-III is the correct explanation for
Statement-II.
(c) Statement-II and Statement-IV are correct, and Statement-II is the correct explanation for
Statement-IV.
(d) Statement-III is correct, but Statement-IV is incorrect
EXPLANATION:
After annexation, the first British land-revenue settlement in Awadh was known as the Summary
Settlement of 1856. It is a more centralized system of revenue collection. This system was introduced by
the British East India Company to gain control over land revenues. This system disrupted the region’s far
older power structure of revenue collection that existed under the control of the taluqdars. This act
undermined the power and position of the taluqdars.
The settlement proceeded on the assumption that the taluqdars were interlopers with no permanent
claims on the land and that they had established their hold through force and fraud. The Summary

SIA-A-GS I (72-A)
Settlement removed the taluqdars wherever possible. The taluqdars of southern Awadh were the hardest
hit and, in certain cases, lost more than half the number of villages they had previously held.
So, Statements I and II are correct.
British land revenue officers believed that by removing taluqdars they would be able to settle the land
with the actual owners of the soil and thus reduce the level of exploitation of peasants while increasing
revenue returns for the state. But this did not happen in practice: revenue flows for the state increased
but the burden of demand on the peasants did not decline. Officials soon found that large areas of Awadh
were actually heavily overassessed, the increase of revenue demand in some places was from 30 to 70 per
cent. Thus neither taluqdars nor peasants had any reasons to be happy with the annexation.
So, Statement III is not correct.
In areas like Awadh, where resistance during 1857 was intense and long-lasting, the fighting was carried
out by taluqdars and their peasants. Many of these taluqdars were loyal to the Nawab of Awadh, and they
joined Begum Hazrat Mahal (the wife of the Nawab) in Lucknow to fight the British; some even remained
with her in defeat.
So, Statement IV is correct.
Thus, Statement II and Statement IV are correct, and Statement II is the correct explanation for
Statement IV. So, Option (c) is correct.

SIA-A-GS I (73-A)

You might also like